ABFM KSA - Diabetes Flashcards

1
Q

Question: 1 of 60

True statements regarding nonpharmacologic therapy to reduce insulin resistance include which of the following? (Mark all that are true.)

  1. Decreasing caloric intake will increase insulin sensitivity independent of weight loss
  2. Moderate alcohol intake increases insulin resistance
  3. Exercise has been shown to enhance insulin action in skeletal muscle
  4. A decrease of as little as 5% in body weight can result in a substantial reduction in insulin resistance
  5. If there are no contraindications, patients with insulin resistance syndrome should be advised to engage in 30 minutes of modest aerobic exercise at least 4–5 times/week
A
  1. Decreasing caloric intake will increase insulin sensitivity independent of weight loss
  2. Moderate alcohol intake increases insulin resistance
  3. Exercise has been shown to enhance insulin action in skeletal muscle
  4. A decrease of as little as 5% in body weight can result in a substantial reduction in insulin resistance
  5. If there are no contraindications, patients with insulin resistance syndrome should be advised to engage in 30 minutes of modest aerobic exercise at least 4–5 times/week

Critique:

Lifestyle interventions play a pivotal role in the management of insulin resistance syndrome. Losing even 5% of body weight has been shown to substantially reduce insulin resistance. In addition, insulin sensitivity can be increased by reducing caloric intake, even if no weight is lost. Exercise is an important adjunct to weight loss, since it has been shown to enhance insulin action in skeletal muscle not only during physical activity but for up to a week following exercise. All patients with insulin resistance syndrome should be advised to engage in 30 minutes of aerobic exercise at least 4–5 times/week. Moderate alcohol intake lowers insulin resistance.

How well did you know this?
1
Not at all
2
3
4
5
Perfectly
2
Q

Question: 2 of 60

Which one of the following neurologic tests is most useful for predicting the future occurrence of a diabetic foot ulcer?

  1. Pressure sensation with Semmes-Weinstein monofilament (10 g)
  2. Deep tendon reflexes of the ankle
  3. Proprioception
  4. Vibratory sensation with a 128-mHz tuning fork
  5. Light touch with a wisp of cotton
A
  1. Pressure sensation with Semmes-Weinstein monofilament (10 g)
  2. Deep tendon reflexes of the ankle
  3. Proprioception
  4. Vibratory sensation with a 128-mHz tuning fork
  5. Light touch with a wisp of cotton

Critique:

Failure to perceive a pressure sensation produced by Semmes-Weinstein monofilament indicates a loss of protective sensation in the diabetic foot and is highly predictive of foot ulceration. Traditional neurologic examination techniques for evaluating reflexes, proprioception, vibration, or light touch are highly subjective and less predictive of future ulceration.

How well did you know this?
1
Not at all
2
3
4
5
Perfectly
3
Q

Question: 3 of 60

Which of the following lipid-lowering agents can worsen glycemic control? (Mark all that are true.)

  1. Colestipol (Colestid)
  2. Ezetimibe (Zetia)
  3. Gemfibrozil (Lopid)
  4. Niacin
  5. Atorvastatin (Lipitor)
A
  1. Colestipol (Colestid)
  2. Ezetimibe (Zetia)
  3. Gemfibrozil (Lopid)
  4. Niacin
  5. Atorvastatin (Lipitor)

Critique:

Niacin is the most effective agent for raising HDL-cholesterol, producing an increase of 15%–35%, it also reduces triglycerides by 20%–50% and LDL-cholesterol by 5%–25%.

Hyperglycemia is a side effect of niacin therapy, particularly at high doses. A dosage of 750–2000 mg/day is associated with only moderate rises in blood glucose, and at one time was considered a treatment option in patients with diabetes, particularly those with low HDL-cholesterol levels. However, the recommendations for niacin use were changed as a result of the AIM-HIGH trial (Atherothrombosis Intervention in Metabolic Syndrome with Low HDL/High Triglycerides: Impact on Global Health Outcomes), which found no incremental clinical benefit from the addition of niacin to statin therapy in patients with coronary heart disease and LDL-cholesterol levels >70 mg/dL.

Recent studies support a link between statin use and the development of diabetes mellitus. In a meta-analysis of 13 studies, statin therapy was associated with a 9% increased risk for incident diabetes. Another meta-analysis corroborated this result and found that intensive-dose statin therapy was associated with a higher risk of new-onset diabetes compared with moderate-dose statin therapy. In 2012, the FDA modified the package labeling of statins to include the risk of increased blood glucose levels and the development of type 2 diabetes. The benefit of statin therapy, however, outweighs the risk; it was estimated there would be 1 additional case of diabetes for every 498 patients treated for 1 year, compared with 1 less patient experiencing a cardiovascular event for every 155 patients treated for 1 year.

How well did you know this?
1
Not at all
2
3
4
5
Perfectly
4
Q

Question: 4 of 60

A 58-year-old male with type 2 diabetes mellitus comes in during the early afternoon for his annual physical examination. His current medication regimen consists of insulin glargine (Lantus), 18 units in the evening; glipizide (Glucotrol), 20 mg/day; metformin (Glucophage), 1000 mg twice a day; and acarbose (Precose), 100 mg three times a day. He suddenly becomes shaky, diaphoretic, and pale, and tells you he thinks it is because he skipped lunch before his appointment.

Which of the following would be effective for managing this episode? (Mark all that are true.)

  1. Glucose tablets
  2. A sugar cube
  3. A banana
  4. A soft drink containing sugar
  5. Raisins
  6. Glucagon
A
  1. Glucose tablets
  2. A sugar cube
  3. A banana
  4. A soft drink containing sugar
  5. Raisins
  6. Glucagon

Critique:

Acarbose, an α-glucosidase inhibitor, inhibits an enzyme present in the brush border of the proximal intestinal epithelium that breaks down disaccharides and more complex carbohydrates. As a result, if hypoglycemia were to occur in a patient on an α-glucosidase inhibitor, reversal requires either the consumption of glucose itself (as opposed to complex carbohydrates) or the injection of glucagon.

How well did you know this?
1
Not at all
2
3
4
5
Perfectly
5
Q

Question: 5 of 60

Which of the following medications can cause hyperglycemia? (Mark all that are true.)

  1. Niacin
  2. Clozapine (Clozaril)
  3. Prednisone
  4. Spironolactone
  5. Ramipril (Altace)
A
  1. Niacin
  2. Clozapine (Clozaril)
  3. Prednisone
  4. Spironolactone
  5. Ramipril (Altace)

Critique:

Several medications have been shown to affect glucose homeostasis, resulting in impaired glucose tolerance and hyperglycemia. Agents associated with the development of hyperglycemia include

  1. pentamidine,
  2. niacin,
  3. glucocorticoids,
  4. thyroid hormone,
  5. diazoxide,
  6. β-adrenergic agonists,
  7. thiazide diuretics,
  8. phenytoin, and
  9. α-interferon.

In addition, second-generation antipsychotic agents, particularly clozapine and olanzapine, have also been linked to the development of hyperglycemia and diabetes mellitus.

Spironolactone and ramipril have not been linked to the development of diabetes. In fact, in the HOPE (Heart Outcomes Prevention Evaluation) study, the use of ramipril, an ACE inhibitor, appeared to reduce the risk for developing type 2 diabetes mellitus by 20%–35%.

How well did you know this?
1
Not at all
2
3
4
5
Perfectly
6
Q

Question: 6 of 60

A 55-year-old African-American male sees you for a routine visit. His past medical history is notable for an 8-year history of diabetes mellitus and a past history of hypercholesterolemia. His current medications are atorvastatin (Lipitor), 20 mg/day, and extended-release metformin (Glucophage XR), 1000 mg/day. He also reports a history of peanut allergy manifested by lip angioedema, and carries an epinephrine auto-injector (EpiPen).

On examination he has a blood pressure of 124/80 mm Hg. His hemoglobin A1c is 6.7%. A spot urine sample contains 40 µg albumin/mg creatinine.

You see the patient 6 months later for a follow-up visit, and a spot urine sample has an albumin/creatinine ratio of 45 µg/mg.

Which one of the following would be most appropriate initially?

  1. Have the patient return in 6 months for a repeat urine test for albumin and creatinine
  2. Order a 24-hour urine collection for creatinine
  3. Recommend that the patient reduce his daily protein intake to 1.5 g/kg/day
  4. Begin an ACE inhibitor
  5. Begin an angiotensin receptor blocker
A
  1. Have the patient return in 6 months for a repeat urine test for albumin and creatinine
  2. Order a 24-hour urine collection for creatinine
  3. Recommend that the patient reduce his daily protein intake to 1.5 g/kg/day
  4. Begin an ACE inhibitor
  5. Begin an angiotensin receptor blocker

Critique:

Diabetic nephropathy develops in 20%–40% of patients with diabetes, and is the leading cause of end-stage renal disease. Persistent albuminuria in the range of 30–200 mg/24 hr (microalbuminuria) is the earliest sign of nephropathy in patients with type 1 diabetes, and is a marker for nephropathy in type 2 diabetes. Patients with microalbuminuria who progress to macroalbuminuria (>300 mg/24 hr) are likely to progress to end-stage renal disease over a period of years.

Although timed 4- and 24-hour urine collections for creatinine can be used to screen for microalbuminuria, a random spot urine specimen for measurement of the albumin-to-creatinine ratio is the preferred method. A minimum of two of three tests showing a urine albumin level >30 µg/mg creatinine or more over a 6-month period confirms the diagnosis of microalbuminuria.

Intensive diabetic management and the use of ACE inhibitors and angiotensin receptor blockers (ARBs) have been shown to delay the progression from microalbuminuria to macroalbuminuria in patients with type 1 or type 2 diabetes. Since the antiproteinuric effect is believed to be independent of blood pressure, current ADA guidelines recommend the use of ACE inhibitors or ARBs as first-line therapy for both type 1 and type 2 diabetic patients with microalbuminuria, even if their blood pressure is normal. Some studies, however, have raised questions about the value of early renin-angiotensin blockade for preventing microalbuminuria in normotensive patients with type 1 or type 2 diabetes, and ADA guidelines recommend against the use of these drugs for patients with normal blood pressure and no albuminuria.

Compared to whites, African-Americans and Asians have a three- to fourfold higher risk of angioedema associated with the use of ACE inhibitors. The American Heart Association recommends that ACE inhibitors not be initiated in any patient with a history of angioedema.

Reduction of protein intake to 0.8–1.0 g/kg/day in the early stages of chronic kidney disease, and to 0.8 g/kg/day in the late stages, may improve renal function and should be considered in patients whose nephropathy seems to be progressive despite optimal glucose and blood pressure control and the use of an ACE inhibitor and/or an ARB.

How well did you know this?
1
Not at all
2
3
4
5
Perfectly
7
Q

Question: 7 of 60

True statements regarding carbohydrate intake and diabetes mellitus include which of the following? (Mark all that are true.)

  1. The glycemic index is not useful in the management of diabetes mellitus
  2. Carbohydrate sources high in protein are effective for treating hypoglycemia
  3. Low-fat diets are more effective for achieving weight loss than low-carbohydrate diets (<130 g/day)
  4. Excessive intake of sugar-sweetened beverages has been shown to increase the risk for diabetes mellitus
  5. Carbohydrates have fewer calories per gram than alcohol
A
  1. The glycemic index is not useful in the management of diabetes mellitus
  2. Carbohydrate sources high in protein are effective for treating hypoglycemia
  3. Low-fat diets are more effective for achieving weight loss than low-carbohydrate diets (<130 g/day)
  4. Excessive intake of sugar-sweetened beverages has been shown to increase the risk for diabetes mellitus
  5. Carbohydrates have fewer calories per gram than alcohol
How well did you know this?
1
Not at all
2
3
4
5
Perfectly
8
Q

Question: 8 of 60

A 51-year-old male with type 2 diabetes mellitus controlled with diet is found to have a serum triglyceride level of 350 mg/dL, an LDL-cholesterol level of 101 mg/dL, and an HDL-cholesterol level of 45 mg/dL.

Which one of the following supplements would most likely reduce his serum triglyceride levels?

  1. Vitamin E
  2. Vitamin C
  3. Omega-3 fatty acids
  4. Folate
  5. Chromium
A
  1. Vitamin E
  2. Vitamin C
  3. Omega-3 fatty acids @ 3 g/day
  4. Folate
  5. Chromium

Critique:

At a dosage of approximately 3 g/day, omega-3 (or n-3) fatty acids have been reported to reduce serum triglyceride concentrations by 25%–30%, with accompanying increases in LDL-cholesterol of 5%–10%, and in HDL-cholesterol of 1%–3%.

How well did you know this?
1
Not at all
2
3
4
5
Perfectly
9
Q

Question: 9 of 60

Which one of the following is INEFFECTIVE for treating pain syndromes arising from diabetic neuropathy?

  1. Tricyclic antidepressants
  2. SSRIs
  3. Duloxetine (Cymbalta)
  4. Pregabalin (Lyrica)
A
  1. Tricyclic antidepressants
  2. SSRIs
  3. Duloxetine (Cymbalta)
  4. Pregabalin (Lyrica)

Critique:

Control of pain represents one of the most challenging management issues in patients with diabetic neuropathy. Tricyclic antidepressants, anticonvulsants, and topical capsaicin have been shown to reduce the pain of diabetic neuropathy.

Pregabalin and duloxetine are both FDA-approved for the treatment of diabetic peripheral neuropathy.

Limited evidence suggests that SSRIs are no more effective than placebo. Although interventions with NSAIDs, transcutaneous electrical nerve stimulation (TENS), ACE inhibitors, and tramadol have been reported for diabetic neuropathy, systematic evaluations have not been published.

How well did you know this?
1
Not at all
2
3
4
5
Perfectly
10
Q

Question: 10 of 60

At a routine health maintenance visit, a 42-year-old obese male is found to have a fasting plasma glucose level of 118 mg/dL. Which one of the following is the most appropriate initial intervention for preventing or delaying the development of diabetes mellitus in this patient?

  1. Lifestyle modification
  2. Metformin (Glucophage)
  3. A thiazolidinedione
  4. An oral sulfonylurea agent
  5. An ACE inhibitor
A
  1. Lifestyle modification
  2. Metformin (Glucophage)
  3. A thiazolidinedione
  4. An oral sulfonylurea agent
  5. An ACE inhibitor

Critique:

Based on the clinical practice guidelines of the American Diabetes Association, impaired fasting glucose (IFG) is defined as a fasting plasma glucose of 100–125 mg/dL, and impaired glucose tolerance (IGT) as a 2-hour plasma glucose of 140–199 mg/dL. These two categories have been officially termed prediabetes and are considered risk factors for future diabetes and cardiovascular disease. Lifestyle modification focusing on weight loss and physical exercise is regarded as first-line therapy for preventing or delaying diabetes mellitus in patients with prediabetes.

In the Diabetes Prevention Program (DPP), lifestyle modification (5%–10% weight loss and moderate physical activity of 30 min/day) was associated with a 58% reduction of risk for developing diabetes.

Metformin can be considered for very high-risk individuals (elevation of both IFG and IGT and at least one other risk factor such as hemoglobin A1C >6%, hypertension, low HDL-cholesterol, elevated serum triglycerides, or family history of type 2 diabetes mellitus in a first degree relative); in the DPP it was associated with a 31% reduction in risk. It was most effective in patients with a BMI of at least 35 kg/m2 who were under age 60.

How well did you know this?
1
Not at all
2
3
4
5
Perfectly
11
Q

Question: 11 of 60

A 77-year-old obese male sees you for a routine visit. He has a 20-year history of hypertension, a 12-year history of type 2 diabetes mellitus complicated by the development of microalbuminuria and proliferative diabetic retinopathy, and a history of an inferior myocardial infarction 2 years ago. Although his diabetes had been adequately controlled with extended-release metformin (Glucophage XR), 500 mg twice daily, you recently added extended-release glipizide (Glucotrol XL), 2.5 mg once daily in the morning, because his hemoglobin A1c rose to 7.1%. He reports that since then he has episodically experienced shakiness and diaphoresis in the late morning, relieved by drinking orange juice. Several of these episodes have occurred during walks he takes with his wife before eating lunch.

Which one of the following would be the most appropriate management?

  1. Reducing his metformin dosage to 500 mg in the morning
  2. Discontinuing glipizide and keeping the patient on his previous drug regimen
  3. Discontinuing glipizide and substituting nateglinide (Starlix)
  4. Advising the patient to eat lunch earlier in the day
  5. Advising the patient to delay his walk until after lunch
A
  1. Reducing his metformin dosage to 500 mg in the morning
  2. Discontinuing glipizide and keeping the patient on his previous drug regimen
  3. Discontinuing glipizide and substituting nateglinide (Starlix)
  4. Advising the patient to eat lunch earlier in the day
  5. Advising the patient to delay his walk until after lunch

Critique:

Although studies have clearly shown that intensive glycemic control reduces the risk for microvascular complications in patients with diabetes, it remains unclear whether it reduces the risk for cardiovascular disease as well. ACCORD, ADVANCE, and the Veterans Affairs Diabetes Trial have failed to show benefit, and the ACCORD trial actually reported an increased mortality rate in patients with type 2 diabetes treated with intensive therapy with a target hemoglobin A1c of <6.0%. Subjects in the ACCORD trial averaged 62 years of age and had diabetes for a mean duration of 10 years. Subjects either had a history of a cardiovascular disease (CVD) event between the ages 40 and 79, or had significant CVD risk and were between the ages 55 and 79. Based on the data available, the American Diabetes Association, in association with the American College of Cardiology Foundation and the American Heart Association, issued a position statement advising that less stringent hemoglobin A1c goals may be appropriate for patients with a history of severe hypoglycemia, limited life expectancy, advanced microvascular or macrovascular complications, poor health, or long-standing diabetes mellitus recalcitrant to therapy. A hemoglobin A1c of <7.0% is still recommended for the majority of patients with diabetes mellitus, with a target hemoglobin A1c closer to normal reserved for healthy patients with a short duration of disease and a long life expectancy (SOR C).

How well did you know this?
1
Not at all
2
3
4
5
Perfectly
12
Q

Question: 12 of 60

True statments regarding dipeptidyl peptidase-4 inhibitors include which of the following? (Mark all that are true.)

  1. They are more effective than metformin for lowering hemoglobin A1c
  2. They reduce insulin resistance
  3. They augment glucagon secretion
  4. They are weight neutral
  5. They are not associated with hypoglycemia
A
  1. They are more effective than metformin for lowering hemoglobin A1c
  2. They reduce insulin resistance
  3. They augment glucagon secretion
  4. They are weight neutral
  5. They are not associated with hypoglycemia

Critique:

Oral dipeptidyl peptidase-4 (DPP-4) inhibitors are oral hypoglycemic agents that work by enhancing circulating concentrations of active glucagon-like peptide 1 (GLP-1) and glucose-dependent insulinotropic peptide (GIP). These agents lower glucose by increasing insulin secretion and reducing glucagon secretion in a glucose-dependent manner. Oral DPP-4 inhibitors are generally felt to be less effective than metformin and the sulfonylureas for lowering glucose, with an expected HbA1c reduction in the range of 0.5%–1.0% compared to 1.0%–1.5% for metformin and the sulfonylureas. DPP-4 inhibitors are considered weight neutral and are not associated with hypoglycemia. Side effects include nasopharyngitis, upper respiratory tract infection, and headache.

How well did you know this?
1
Not at all
2
3
4
5
Perfectly
13
Q

Question: 13 of 60

A 62-year-old male sees you for the first time. His past medical history is notable for a long history of type 2 diabetes and hypertension, as well as a history of myocardial infarction 5 years ago and New York Heart Association class III heart failure. His current medications are hydrochlorothiazide, 25 mg daily; valsartan (Diovan), 320 mg daily; metoprolol succinate (Toprol XL), 50 mg daily, metformin (Glucophage), 850 mg twice daily; rosuvastatin (Crestor), 20 mg daily; and aspirin, 81 mg daily. Notable findings on examination include a blood pressure of 135/84 mm Hg and a heart rate of 58 beats/min. Laboratory findings include a hemoglobin A1C of 7.8%, an LDL-cholesterol level of 70 mg/dL, an HDL-cholesterol level of 35 mg/dL, a serum triglyceride level of 210 mg/dL, and an estimated glomerular filtration rate of 71 mL/min/1.73 m2.

Which one of the following has been shown to reduce cardiovascular risk in patients such as this?

  1. Glipizide extended-release (Glucotrol XL)
  2. Liraglutide (Victoza)
  3. Niacin
  4. Pioglitazone (Actos)
  5. Saxagliptin (Onglyza)
A
  1. Glipizide extended-release (Glucotrol XL)
  2. Liraglutide (Victoza) – only FDA approved hypoglycemic agent for LOWERING risk of cardiovascular event.
  3. Niacin
  4. Pioglitazone (Actos)
  5. Saxagliptin (Onglyza)

Critique:

The LEADER trial (Liraglutide Effect and Action in Diabetes: Evaluation of Cardiovascular Outcome Results) was a double-blind trial that compared the use of liraglutide, a GLP-1 analogue, to placebo in 9340 patients with type 2 diabetes at high cardiovascular risk. After a mean follow-up of 3.8 years, liraglutide was found to significantly reduce the rate of death from cardiovascular causes, as well as the first occurrence of nonfatal myocardial infarction and nonfatal stroke (hazard ratio, 0.87; 95% confidence interval [CI], 0.78 to 0.97). The rate of death from any cause was also reduced (SOR B).

Cardiovascular outcome studies evaluating DPP-4 inhibitors such as sitagliptin, saxagliptin, and alogliptin have yet to demonstrate a significant reduction in adverse cardiovascular events in patients with diabetes. In addition, the SAVOR-TIMI trial (Saxagliptin Assessment of Vascular Outcomes Recorded in Patients with Diabetes Mellitus—TIMI 53) reported a higher risk for hospitalization for heart failure associated with saxagliptin treatment compared to placebo. Although a reduced risk for stroke has been reported with pioglitazone, thiozolidenediones are associated with fluid retention, which can lead to weight gain, edema, and heart failure. Their use is contraindicated in patients with New York Heart Association class III or IV heart failure. Oral sulfonylureas are potent glucose-lowering agents associated with a higher risk for hypoglycemia. Studies have not shown a reduced cardiovascular risk with their use, and the University Group Diabetes Program (UGDP) trial reported a higher risk of cardiovascular death associated with the use of tolbutamide.

Although niacin might have been a consideration in the past in an effort to raise HDL-cholesterol and lower triglycerides, support for its use was dampened by the findings of the AIM-HIGH trial (Atherothrombosis Intervention in Metabolic Syndrome with Low HDL/High Triglycerides: Impact on Global Health Outcomes), which found no clinical benefit from adding sustained-release niacin to a statin in patients with known coronary heart disease and low HDL-cholesterol.

How well did you know this?
1
Not at all
2
3
4
5
Perfectly
14
Q

Question: 14 of 60

A 66-year-old male who was hospitalized because of a TIA 3 months ago sees you for a follow-up visit. His past medical history is notable for impaired fasting glucose and mild hypertension. His current medications are valsartan (Diovan), 160 mg daily; rosuvastatin (Crestor), 20 mg daily; and aspirin, 81 mg daily. On examination his BMI is 30.2 kg/m2, his blood pressure is 134/86 mm Hg, and he has brown, velvety, hyperkeratotic plaques on the back of his neck and in his axilla. His laboratory studies are notable for an LDL-cholesterol level of 85 mg/dL, an HDL-cholesterol level of 35 mg/dL, and a serum triglyceride level of 174 mg/dL.

Which one of the following agents may reduce his risk for stroke and myocardial infarction?

  1. Basal insulin
  2. Acarbose (Precose)
  3. Glipizide (Glucotrol)
  4. Pioglitazone (Actos)
  5. Sitagliptin (Januvia)
A
  1. Basal insulin
  2. Acarbose (Precose)
  3. Glipizide (Glucotrol)
  4. Pioglitazone (Actos)
  5. Sitagliptin (Januvia)

Critique:

In addition to playing a primary role in the development of type 2 diabetes, insulin resistance is also found in more than half of patients without diabetes who experience an ischemic stroke or TIA. Although treatment of individual cardiovascular risk factors plays a major role in the management of these patients, treatment directly targeted at reducing insulin resistance may also have a role.

The Insulin Resistance Intervention in Stroke (IRIS) trial was a 4.8-year multicenter double-blind study that investigated the role of pioglitazone in nondiabetic stroke and TIA patients determined to have insulin resistance based on the homeostasis model assessment of insulin resistance (HOMA-IR) index.

  • In this trial, the use of pioglitazone was associated with a 24% reduction in stroke and myocardial infarction and a 52% reduction in the risk of developing type 2 diabetes.

This potential benefit should be balanced against possible adverse events linked to thiazolidinedione use, including weight gain, edema, bone fracture, and bladder cancer.

How well did you know this?
1
Not at all
2
3
4
5
Perfectly
15
Q

Question: 15 of 60

Hypoglycemia is a possible side effect of which of the following diabetes agents? (Mark all that are true.)

  1. Insulin
  2. Pioglitazone (Actos)
  3. Metformin (Glucophage)
  4. Sulfonylureas
  5. Repaglinide (Prandin)
  6. Acarbose (Precose)
A
  1. Insulin
  2. Pioglitazone (Actos)
  3. Metformin (Glucophage)
  4. Sulfonylureas
  5. Repaglinide (Prandin)
  6. Acarbose (Precose)

Critique:

Hypoglycemia is a well-known complication of insulin therapy. Since sulfonylureas (SU’s) and repaglinide (Meglitinide) work by enhancing insulin secretion, hypoglycemia is a complication of these two agents as well.

Used alone, acarbose, metformin, and thiazolidinediones (e.g., pioglitazone) are not associated with the development of hypoglycemia. It should be noted, however, that in June 2011, the FDA issued a drug safety alert reporting that use of pioglitazone for more than 1 year may be associated with an increased risk of bladder cancer.

How well did you know this?
1
Not at all
2
3
4
5
Perfectly
16
Q

Question: 16 of 60

True statements regarding dietary fat intake in patients with diabetes mellitus include which of the following? (Mark all that are true.)

  1. A Mediterranean-style diet rich in monounsaturated fats has been shown to improve glycemic control in patients with diabetes
  2. Trans fatty acids have been shown to lower LDL-cholesterol and raise HDL-cholesterol
  3. Saturated fats should provide 10% of caloric intake
  4. Omega-3 (or n-3) fatty acid supplementation is associated with a cardioprotective effect
  5. A gram of fat contains 50% more calories than a gram of carbohydrate
A
  1. A Mediterranean-style diet rich in monounsaturated fats has been shown to improve glycemic control in patients with diabetes
  2. Trans fatty acids have been shown to lower LDL-cholesterol and raise HDL-cholesterol
  3. Saturated fats should provide 10% of caloric intake
  4. Omega-3 (or n-3) fatty acid supplementation is associated with a cardioprotective effect
  5. A gram of fat contains 50% more calories than a gram of carbohydrate

Critique:

The primary goal with regard to fat intake in patients with diabetes is to limit saturated fat and trans fatty acids.

  • National dietary guidelines recommend that intake of saturated fat be limited to <10% of daily calories.

Intake of trans unsaturated fatty acids should be minimized, since they have been shown to raise LDL-cholesterol and lower HDL-cholesterol.

A Mediterranean-style diet rich in monunsaturated fats has been found to improve both glycemic control and lipid levels in patients with diabetes.

Randomized, controlled trials do NOT support recommending omega-3 supplements for primary or secondary prevention of cardiovascular disease.

A gram of fat contains more than twice the calories of a gram of carbohydrate.

How well did you know this?
1
Not at all
2
3
4
5
Perfectly
17
Q

Question: 17 of 60

True statements regarding coronary heart disease in patients with diabetes mellitus include which of the following? (Mark all that are true.)

  1. Routine screening with a cardiac stress test is recommended in asymptomatic patients with diabetes who are at increased cardiovascular risk
  2. β-Blockers should be avoided in diabetic patients with coronary artery disease, due to the risk of masking hypoglycemia and reducing insulin secretion
  3. Long-term outcomes following percutaneous transluminal coronary angioplasty are as good in diabetic patients as in nondiabetic patients
  4. The survival of diabetic patients with multivessel disease is better with coronary revascularization with coronary artery bypass graft (CABG) surgery than with percutaneous transluminal coronary angioplasty
  5. Optimal glycemic control has been shown to reduce the risk of coronary heart disease in patients with type 2 diabetes
A
  1. Routine screening with a cardiac stress test is recommended in asymptomatic patients with diabetes who are at increased cardiovascular risk
  2. β-Blockers should be avoided in diabetic patients with coronary artery disease, due to the risk of masking hypoglycemia and reducing insulin secretion
  3. Long-term outcomes following percutaneous transluminal coronary angioplasty are as good in diabetic patients as in nondiabetic patients
  4. The survival of diabetic patients with multivessel disease is better with coronary revascularization with coronary artery bypass graft (CABG) surgery than with percutaneous transluminal coronary angioplasty
  5. Optimal glycemic control has been shown to reduce the risk of coronary heart disease in patients with type 2 diabetes

Critique:

Although atherosclerotic cardiovascular disease is the leading cause of morbidity and mortality in patients with diabetes, routine screening for coronary heart disease is NOT recommended since it has not been shown to improve cardiovascular outcomes provided cardiovascular risk factors are treated (SOR A).

The potential benefit of β-blockers in the diabetic patient with coronary artery disease outweighs the potential risk of masking hypoglycemia or reducing insulin secretion (SOR A).

Good glycemic control has been shown to reduce microvascular complications in patients with diabetes mellitus. Although the Diabetes Control and Complications Trial and the Epidemiology of Diabetes Interventions and Complications study found that intensive glycemic control initiated soon after the diagnosis of type 1 diabetes produced long-term protection from cardiovascular disease, the results of three large trials (ACCORD, ADVANCE, and VADT) published in 2008 suggested no reduction in cardiovascular disease risk with intensive glycemic control in patients with type 2 diabetes.

Mortality rates after percutaneous transluminal coronary angioplasty (PTCA) are generally higher in patients with diabetes mellitus than in nondiabetic patients.

The survival of diabetic patients with multivessel disease is better after coronary artery bypass graft (CABG) surgery than after PTCA. This was shown in the FREEDOM trial (Future Revascularization Evaluation in Patients with Diabetes Mellitus: Optimal Management of Multivessel Disease), a randomized trial of 1900 patients with diabetes and multivessel coronary heart disease.

Treatment with CABG was associated with both a lower rate of myocardial infarction and lower mortality compared to PCI with drug-eluting stents (SOR A).

How well did you know this?
1
Not at all
2
3
4
5
Perfectly
18
Q

Question: 18 of 60

The threshold fasting plasma glucose level recommended for confirming the diagnosis of diabetes mellitus is ________ mg/dL

A

Fast BSL 126 mg/dL (or HA1c > 6.5)

Critique:

According to the current criteria of the American Diabetes Association, the diagnosis of diabetes mellitus should be made if any one of the following criteria is met:

  • fasting plasma glucose ≥126 mg/dL
  • 2-hour post load glucose ≥200 mg/dL on oral glucose tolerance testing
  • hemoglobin A1c ≥6.5%
  • symptoms of diabetes plus a casual plasma glucose level ≥200 mg/dL
How well did you know this?
1
Not at all
2
3
4
5
Perfectly
19
Q

Question: 19 of 60

A 62-year-old African-American male with a 10-year history of type 2 diabetes is diagnosed with hypertension. His current medications include metformin (Glucophage XR), 1500 mg daily; sitagliptin (Januvia), 100 mg daily; and simvastatin (Zocor), 40 mg daily. His blood pressure at today’s visit is 154/94 mm Hg. His urine is negative for microalbuminuria.

Which one of the following is true regarding treatment recommendations for this patient?

  1. Current American Diabetes Association (ADA) guidelines recommend treatment to a systolic blood pressure goal of <130 mm Hg and a diastolic blood pressure goal of <85 mm Hg
  2. JNC 8 guidelines recommend treatment to a systolic blood pressure goal <150 mm Hg and a diastolic blood pressure goal of <90 mm Hg
  3. The SPRINT trial supports targeting a systolic blood pressure goal of <120 mm Hg
  4. ADA guidelines recommend initiating therapy with either an ACE inhibitor or an angiotensin receptor blocker
  5. JNC 8 guidelines recommend initiating therapy with either a thiazide-like diuretic or a dihydropyridine calcium channel blocker
A
  1. Current American Diabetes Association (ADA) guidelines recommend treatment to a systolic blood pressure goal of <130 mm Hg and a diastolic blood pressure goal of <85 mm Hg
  2. JNC 8 guidelines recommend treatment to a systolic blood pressure goal <150 mm Hg and a diastolic blood pressure goal of <90 mm Hg
  3. The SPRINT trial supports targeting a systolic blood pressure goal of <120 mm Hg
  4. ADA guidelines recommend initiating therapy with either an ACE inhibitor or an angiotensin receptor blocker
  5. JNC 8 guidelines recommend initiating therapy with either a thiazide-like diuretic or a dihydropyridine calcium channel blocker

Critique:

Although randomized clinical trials have shown the cardiovascular and renal benefit of antihypertensive treatment targeting a systolic blood pressure <140 mm Hg and a diastolic blood pressure <90 mm Hg, they have not generally demonstrated additional benefit with more intensive therapy (targeting a systolic blood pressure <120–30 mm Hg and a diastolic blood pressure <80 mm Hg) in patients with diabetes.

JNC 8 guidelines recommend a target systolic blood pressure of <150 mm Hg and a target diastolic blood pressure of <90 mm Hg in individuals over 60 years of age, but the guidelines recommend a target systolic blood pressure of <140 mm Hg in individuals over 18 years of age with diabetes mellitus (SOR C).

Current American Diabetes Association (ADA) guidelines generally recommend a target systolic blood pressure of <140 mm Hg and a target diastolic blood pressure of <90 mm Hg (SOR A), with lower targets, such as 130/80 mm Hg, for high-risk patients with diabetes if these goals are achievable without undue treatment burden (SOR C). The 2017 ACC/AHA hypertension guidelines take a more aggressive approach, recommending antihypertensive drug treatment be initiated at a blood pressure of 130/80 mm Hg or higher, with a treatment goal of <130/80 mm Hg in patients with diabetes.

With the exception of the diabetic patients with albuminuria, ADA guidelines recommend initiating therapy with any of the antihypertensive agents shown to reduce cardiovascular events in patients with diabetes, which includes ACE inhibitors, angiotensin receptor blockers (ARBs), thiazide-like diuretics, and dihydropyridine calcium channel blockers. First-line treatment with an ACE inhibitor or ARB is recommended by the ADA for diabetic individuals with albuminuria (SOR B). In the general African-American population, including those with diabetes, JNC 8 guidelines favor initial treatment with a thiazide-like diuretic or dihydropyridine calcium channel blocker; however, in patients with chronic kidney disease, regardless of race or diabetes status, both JNC 8 and the ADA recommend initial therapy with an ACE inhibitor or ARB.

SPRINT (Systolic Blood Pressure Intervention Trial) was a randomized, controlled, open-label trial that compared aggressive treatment to a target systolic blood pressure <120 mm Hg with a target of <140 mm Hg in patients at increased cardiovascular risk. Although it did find that targeting a systolic blood pressure of <120 mm Hg resulted in lower rates of fatal and nonfatal major cardiovascular events, patients with diabetes or history of previous stroke were specifically excluded from the study.

How well did you know this?
1
Not at all
2
3
4
5
Perfectly
20
Q

Question: 20 of 60

Endocrinopathies associated with diabetes mellitus include which of the following? (Mark all that are true.)

  1. Cushing’s syndrome
  2. Acromegaly
  3. Pheochromocytoma
  4. Gastrinoma
  5. Glucagonoma
A
  1. Cushing’s syndrome
  2. Acromegaly
  3. Pheochromocytoma
  4. Gastrinoma
  5. Glucagonoma

Critique:

Endogenous gluconeogenic hormones include cortisol, norepinephrine, epinephrine, glucagon, and growth hormone. Accordingly, endocrinopathies associated with excessive levels of these hormones can cause glucose intolerance and diabetes; such disorders include

  1. acromegaly,
  2. Cushing’s syndrome,
  3. glucagonoma, and
  4. pheochromocytoma.
  5. Hyperthyroidism has also been shown to be associated with diabetes mellitus. In addition,
  6. somatostatinomas and
  7. aldosteronomas can cause diabetes, most likely by inhibiting insulin secretion.
How well did you know this?
1
Not at all
2
3
4
5
Perfectly
21
Q

Question: 21 of 60

According to National Cholesterol Education Program guidelines, criteria for the diagnosis of metabolic syndrome include which of the following? (Mark all that are true.)

  1. A waist circumference >40 inches in males
  2. An HDL-cholesterol level <50 mg/dL in females
  3. An LDL-cholesterol level ≥160 mg/dL
  4. A serum triglyceride level ≥150 mg/dL
  5. Diastolic blood pressure ≥85 mm Hg
A
  1. A waist circumference >40 inches in males
  2. An HDL-cholesterol level <50 mg/dL in females
  3. An LDL-cholesterol level ≥160 mg/dL
  4. A serum triglyceride level ≥150 mg/dL
  5. Diastolic blood pressure ≥85 mm Hg

Critique:

Metabolic syndrome is a constellation of cardiovascular risk factors related to hypertension, abdominal obesity, dyslipidemia, and insulin resistance.

Diagnostic criteria for metabolic syndrome, according to the National Cholesterol Education Program (Adult Treatment Panel III Guidelines), include the presence of three or more of the following:

  1. obesity, with a waist circumference exceeding 102 cm (40 inches) in men or 88 cm (35 inches) in women;
  2. blood pressure ≥130 mm Hg systolic and/or 85 mm Hg diastolic;
  3. a fasting glucose level ≥110 mg/dL;
  4. a serum triglyceride level ≥150 mg/dL; and
  5. an HDL-cholesterol level <40 mg/dL in men or <50 mg/dL in women.
How well did you know this?
1
Not at all
2
3
4
5
Perfectly
22
Q

Question: 22 of 60

A 62-year-old female is diagnosed with type 2 diabetes mellitus on the basis of consecutive fasting plasma glucose levels of 138 mg/dL and 143 mg/dL. Current American Diabetes Association guidelines recommend which of the following as part of her initial management? (Mark all that are true.)

  1. Lifestyle intervention
  2. Metformin (Glucophage)
  3. An oral sulfonylurea
  4. A thiazolidinedione
  5. Pramlintide (Symlin)
A
  1. Lifestyle intervention
  2. Metformin (Glucophage)
  3. An oral sulfonylurea
  4. A thiazolidinedione
  5. Pramlintide (Symlin)

Critique:

Most individuals with type 2 diabetes mellitus fail to achieve or maintain metabolic goals with only lifestyle intervention, because of failure to lose or maintain weight loss, progressive disease, or a combination of factors. As a result, current American Diabetes Association treatment guidelines recommend that metformin be initiated concurrently with lifestyle intervention at the time of diagnosis of type 2 diabetes mellitus.

How well did you know this?
1
Not at all
2
3
4
5
Perfectly
23
Q

Question: 23 of 60

Pharmacologic agents found to be effective in reducing the progression of impaired glucose tolerance to overt diabetes include which of the following? (Mark all that are true.)

  1. Acarbose (Precose)
  2. Metformin (Glucophage)
  3. Repaglinide (Prandin)
  4. Pioglitazone (Actos)
  5. Orlistat (Alli, Xenical)
A
  1. Acarbose (Precose)
  2. Metformin (Glucophage)
  3. Repaglinide (Prandin)
  4. Pioglitazone (Actos)
  5. Orlistat (Alli, Xenical)

Critique:

There is evidence that a number of pharmacologic interventions may be of value in preventing the development of type 2 diabetes mellitus in patients with impaired glucose tolerance (or prediabetes). Drug therapy with

  • metformin (a biguanide),
  • orlistat (a lipase inhibitor),
  • GLP-1 receptor agonists, or
  • acarbose (an α-glucosidase inhibitor)

has been shown to delay or prevent the progression of impaired glucose tolerance to type 2 diabetes.

The ACT NOW study reported a reduction of incident type 2 diabetes in patients with impaired glucose tolerance or impaired fasting glucose treated with pioglitazone. In addition, the ACT NOW study recently reported that compared to placebo, pioglitazone reduced the risk of conversion of impaired glucose tolerance to type 2 diabetes by 72%.

It should be noted, however, that in June 2011, the FDA issued a drug safety alert reporting that use of pioglitazone for more than 1 year may be associated with an increased risk of bladder cancer. After factoring in cost, side effects, and evidence of long-term effect, American Diabetes Association guidelines recommended metformin as the only drug for use in diabetes prevention.

How well did you know this?
1
Not at all
2
3
4
5
Perfectly
24
Q

Question: 24 of 60

A 29-year-old female with polycystic ovary syndrome (PCOS) asks if you can correct her oligomenorrhea. Her fasting glucose level is 100 mg/dL and her hemoglobin A1c is in the desirable range.

Which one of the following diabetes medications would be most appropriate for managing her oligomenorrhea?

  1. Glyburide (DiaBeta)
  2. Metformin (Glucophage)
  3. Pioglitazone (Actos)
  4. Miglitol (Glyset)
  5. Repaglinide (Prandin)
A
  1. Glyburide (DiaBeta)
  2. Metformin (Glucophage)
  3. Pioglitazone (Actos)
  4. Miglitol (Glyset)
  5. Repaglinide (Prandin)

Critique:

Polycystic ovary syndrome affects an estimated 6% of women of reproductive age. Insulin resistance with compensatory hyperinsulinemia is thought to play a major role in the etiology of this syndrome, which is characterized by anovulation and hyperandrogenism. Metformin and thiazolidinediones (i.e., pioglitazone and rosiglitazone) are oral hypoglycemic agents that reduce insulin resistance and have been shown to increase the frequency of ovulation in patients with this syndrome.

Caution is advised with the use of thiazolidenediones, however. In addition to troglitazone being withdrawn from the market, a meta-analysis of 42 studies found a 43% increased risk of myocardial infarction in patients taking rosiglitazone compared with other antidiabetic agents. This increased risk was subsequently corroborated by another published meta-analysis. In addition, the FDA issued a drug safety alert in 2011 reporting that use of pioglitazone for more than 1 year may be associated with an increased risk of bladder cancer. However, in November 2013 the FDA determined that more recent data indicates that rosiglitazone-containing drugs do not increase the risk of heart attack when compared to metformin and sulfonylurea, and removed the prescribing and dispensing restrictions.

How well did you know this?
1
Not at all
2
3
4
5
Perfectly
25
Q

Question: 25 of 60

A 53-year-old obese male with a history of type 2 diabetes mellitus sees you for the first time. He tells you that his previous physician had him see a dietician and started him on metformin (Glucophage), 500 mg twice daily. He brings in a copy of his most recent laboratory tests and you note that his hemoglobin A1c is 7.7%. He admits he has always been sedentary, and wonders if it would be worthwhile for him to join an exercise facility and begin an exercise program.

Which of the following would be appropriate advice? (Mark all that are true.)

  1. Aerobic exercise can be expected to lower hemoglobin A1c by at least 1 percentage point
  2. Resistance training has been shown to improve glycemic control
  3. Combined aerobic and resistance training results in greater glycemic improvement than either method alone
  4. A minimum of 150 minutes of moderate-intensity exercise per week is recommended
  5. Improved glycemic control is seen only in those who exercise and achieve a reduction in BMI
A
  1. Aerobic exercise can be expected to lower hemoglobin A1c by at least 1 percentage point
  2. Resistance training has been shown to improve glycemic control
  3. Combined aerobic and resistance training results in greater glycemic improvement than either method alone
  4. A minimum of 150 minutes of moderate-intensity exercise per week is recommended
  5. Improved glycemic control is seen only in those who exercise and achieve a reduction in BMI

Critique:

Both aerobic training and resistance training have both been shown to improve glycemic control.

In a randomized, controlled trial, an aerobic exercise program consisting of three exercise sessions per week, consisting of up to 45 minutes per session at 70%–75% of the maximum predicted heart rate (determined on a treadmill exercise test), reduced hemoglobin A1c by 0.5 percentage points.

A resistance training program performed three times per week improved hemoglobin A1c by 0.38 percentage points. Combined exercise training produced an additional improvement of 0.46 percentage points compared to aerobic training alone, and an improvement of 0.59 percentage points compared to resistance training alone.

Notably, in patients with a baseline hemoglobin A1c <7.5%, significant glycemic improvement was observed only in those in the combined exercise training groups. The American Heart Association recommends that patients with type 2 diabetes mellitus accumulate a minimum of 150 minutes per week of moderate-intensity exercise and/or 90 minutes per week of vigorous-intensity cardiorespiratory exercise. Structured exercise interventions have been shown to lower hemoglobin A1c even with no significant change in BMI.

26
Q

Question: 26 of 60

A 35-year-old male sees you for a routine health maintenance visit. He admits he has gained a few pounds over the past few years. He is 173 cm (68 in) tall and weighs 82 kg (181 lb), giving him a BMI of 27.3 kg/m2.

According to current American Diabetes Association guidelines, which of the following would warrant screening for prediabetes and diabetes? (Mark all that are true.)

  1. An LDL-cholesterol level >160 mg/dL
  2. A serum triglyceride level >250 mg/dL
  3. A family history of diabetes mellitus in his father
  4. Blood pressures in the prehypertensive range at his last two office visits
  5. A history of physical inactivity
A
  1. An LDL-cholesterol level >160 mg/dL
  2. A serum triglyceride level >250 mg/dL
  3. A family history of diabetes mellitus in his father
  4. Blood pressures in the prehypertensive range at his last two office visits
  5. A history of physical inactivity

Critique:

The estimated prevalence of diabetes mellitus among adults was 7.4% in 1995 and is projected to rise to 9.0% by 2025.

Screening is recommended in asymptomatic adults beginning at age 45 in the absence of risk factors, and should be repeated at least once every 3 years.

Screening should be considered at an earlier age in patients who have a BMI ≥25.0 kg/m2 and additional risk factors, including physical inactivity, a first degree relative with diabetes mellitus, a history of gestational diabetes or of delivering an infant weighing more than 9 lb, belonging to certain racial/ethnic groups (e.g., Native Americans, African-Americans, Hispanics, Asians, South Pacific Islanders), hypertension, an HDL-cholesterol level <35 mg/dL, a serum triglyceride level >250 mg/dL, polycystic ovary syndrome, impaired fasting glucose or impaired glucose tolerance on previous testing, a history of cardiovascular disease, or other conditions associated with insulin resistance (e.g., acanthosis nigricans, severe obesity).

27
Q

Question: 27 of 60

Which one of the following is true regarding SLGT2 inhibitors?

  1. They lower glucose by reducing glucagon secretion
  2. Studies have shown that they reduce cardiovascular risk
  3. Studies have not demonstrated a renoprotective effect
  4. They have been linked to a higher risk of heart failure
  5. Adverse effects include a higher risk for sinus infections and genital virus infections
A
  1. They lower glucose by reducing glucagon secretion
  2. Studies have shown that they reduce cardiovascular risk
  3. Studies have not demonstrated a renoprotective effect
  4. They have been linked to a higher risk of heart failure
  5. Adverse effects include a higher risk for sinus infections and genital virus infections

Critique:

SGLT2 inhibitors (Canagli-flozin, Dapagli-flozin) improve glucose control by preventing reabsorption of glucose and increasing urinary glucose excretion.

Clinical studies have shown that these agents lead to a significant reduction in albuminuria and a slower decline in glomerular filtration rate (GFR), as well as a reduction in cardiovascular risk. The EMPA-REG OUTCOME study demonstrated a 14% relative risk reduction for the primary composite 3-point MACE outcome of cardiovascular death, nonfatal myocardial infarction, and nonfatal stroke in patients with type 2 diabetes who were at high cardiovascular risk and were receiving empagliflozin in addition to standard care. These patients also had a 35% reduction in risk for hospitalization for heart failure (SOR B). With regard to renal outcomes, the EMPA-REG OUTCOME study found that patients receiving empagliflozin had a slower progression of kidney disease and lower rates of clinically relevant renal events.

The CANVAS (Canagliflozin Cardiovascular Assessment Study) program, which integrated the results of two double-blind, placebo-controlled trials (CANVAS and CANVAS-R), reported similar benefits with the use of canagliflozin in patients with type 2 diabetes. In this study, the SGLT2 inhibitor canagliflozin reduced the primary composite outcome of cardiovascular death, nonfatal myocardial infarction, and nonfatal stroke by 14% and reduced the rate of renal decline by 40% among patients with type 2 diabetes who were at high cardiovascular risk. Adverse events seen with SGLT2 inhibitors include genital fungal infections, urinary tract infections, and volume depletion. In addition, the CANVAS trials also reported a higher risk for amputation and fracture. Euglycemic diabetic ketoacidosis has also been linked to the use of SGLT2 inhibitors. In 2015 the FDA issued a Drug Safety Communication regarding all of the currently available SGLT2 inhibitors, warning of a higher risk of diabetic ketoacidosis with uncharacteristically mild or moderate glucose elevations.

28
Q

Question: 28 of 60

A BMI ≥23.0 kg/m2 is a risk factor for diabetes in which one of the following ethnic groups?

  1. African-American
  2. Asian-American
  3. Native American
  4. Hispanic/Latino
  5. Non-Hispanic white
A
  1. African-American
  2. Asian-American
  3. Native American
  4. Hispanic/Latino
  5. Non-Hispanic white

Critique:

Ethnic backgrounds associated with a higher risk for developing type 2 diabetes include

  • Native American,
  • African-American,
  • Hispanic/Latino,
  • Asian-American, and
  • Pacific Islander.

The American Diabetes Association generally recommends screening for prediabetes and diabetes mellitus in adults belonging to one of these racial/ethnic groups if they have a BMI >25.0 kg/m2. However, epidemiologic studies have shown the relationship between BMI and diabetes risk in Asian-Americans is associated with lower BMI values than other racial groups. Possible reasons for this include differences in body fat distribution (Asians may have a greater propensity to develop visceral versus peripheral adiposity) as well as body composition (Asian-Americans may have a higher percentage of body fat at any given BMI level). As a result, the American Diabetes Association lowered the BMI threshold to 23.0 kg/m2 for defining increased risk for diabetes in Asian-Americans (SOR B).

29
Q

Question: 29 of 60

Which of the following oral agents should be used with caution in patients with advanced heart failure? (Mark all that are true.)

  1. Thiazolidinediones
  2. Metformin (Glucophage)
  3. Sulfonylureas
  4. Meglitinides
  5. α-Glucosidase inhibitors
A
  1. Thiazolidinediones
  2. Metformin (Glucophage)
  3. Sulfonylureas
  4. Meglitinides
  5. α-Glucosidase inhibitors

Critique:

Because of their propensity to expand plasma volume, thiazolidinediones should be used only with caution in patients with New York Heart Association class II heart failure and are contraindicated in patients with class III or IV heart failure.

Patients with advanced heart failure are at higher risk for developing lactic acidosis and renal impairment, and metformin should consequently be used with caution in such patients.

30
Q

Question: 30 of 60

Clinical conditions associated with insulin resistance syndrome include which of the following? (Mark all that are true.)

  1. Atherosclerotic cardiovascular disease
  2. Polycystic ovary syndrome
  3. Acanthosis nigricans
  4. Nonalcoholic steatohepatitis
  5. Polycythemia
A
  1. Atherosclerotic cardiovascular disease
  2. Polycystic ovary syndrome
  3. Acanthosis nigricans
  4. Nonalcoholic steatohepatitis
  5. Polycythemia

Critique:

Medical conditions that increase the likelihood of insulin resistance syndrome include the presence of cardiovascular disease, hypertension, polycystic ovary syndrome, acanthosis nigricans, and nonalcoholic fatty liver disease.

31
Q

Question: 31 of 60

True statements regarding aspirin therapy in patients with diabetes mellitus include which of the following? (Mark all that are true.)

  1. The recommended dosage is 75–162 mg/day
  2. It is recommended for routine use in diabetic patients over 40 years of age
  3. It is recommended for teenage diabetic patients with dyslipidemia
  4. Its use in patients under 21 years of age is associated with an increased risk of Reye’s syndrome
  5. It is recommended for all adults with microalbuminuria
A
  1. The recommended dosage is 75–162 mg/day
  2. It is recommended for routine use in diabetic patients over 40 years of age
  3. It is recommended for teenage diabetic patients with dyslipidemia
  4. Its use in patients under 21 years of age is associated with an increased risk of Reye’s syndrome
  5. It is recommended for all adults with microalbuminuria

Critique:

Although aspirin has been shown to be effective for secondary prevention in high-risk patients with known coronary heart disease or a history of previous stroke, its net benefit as a primary preventive measure is more controversial. Previous American Diabetes Association (ADA) guidelines have recommended that aspirin be routinely prescribed for diabetic patients older than 40 years of age, but recent clinical trials of aspirin in patients with diabetes mellitus have failed to demonstrate significant cardiovascular benefit.

Current ADA guidelines recommend that low-dose (75–162 mg/day) aspirin use be considered as a primary prevention strategy in those with type 1 or type 2 diabetes who are at increased cardiovascular risk.

This includes most men and women with diabetes who are over 50 years of age and have at least one additional major risk factor (family history of premature atherosclerotic cardiovascular disease, hypertension, dyslipidemia, smoking, or albuminuria) and are not at increased risk of bleeding (SOR C).

Aspirin therapy is generally not recommended in patients under the age of 21 years because of an increased risk of Reye’s syndrome in this population.

32
Q

Question: 32 of 60

True statements regarding diabetic retinopathy include which of the following? (Mark all that are true.)

  1. Laser photocoagulation therapy has not been shown to be of benefit in patients with macular edema
  2. Retinopathy is a contraindication to aspirin therapy
  3. Glycemic control has been shown to prevent and delay progression of retinopathy
  4. There is no evidence that blood pressure control has a favorable impact on the progression of diabetic retinopathy
  5. Panretinal photocoagulation has been shown to reduce severe visual loss in patients with proliferative retinopathy
A
  1. Laser photocoagulation therapy has not been shown to be of benefit in patients with macular edema
  2. Retinopathy is a contraindication to aspirin therapy
  3. Glycemic control has been shown to prevent and delay progression of retinopathy
  4. There is no evidence that blood pressure control has a favorable impact on the progression of diabetic retinopathy
  5. Panretinal photocoagulation has been shown to reduce severe visual loss in patients with proliferative retinopathy

Critique:

The management of diabetic retinopathy calls for control of medical factors known to affect retinopathy and its progression. The Diabetes Control and Complications Trial (DCCT) demonstrated that tight glycemic control favorably affected the onset and progression of diabetic retinopathy. High blood pressure is an established risk factor for the development of macular edema and is associated with the presence of proliferative diabetic retinopathy; lowering blood pressure was shown by the UKPDS study to reduce the risk of progression. There is no evidence that aspirin therapy has a favorable effect on retinopathy. The presence of retinopathy is not a contraindication to aspirin therapy for cardioprotection since this therapy has not been shown to increase the risk of retinal hemorrhage. Panretinal photocoagulation has been shown to reduce severe visual loss in patients with diabetic retinopathy, particularly in those with disc revascularization or vitreous hemorrhage with any retinal revascularization. The Early Treatment Diabetic Retinopathy Study demonstrated the benefit of focal laser photocoagulation surgery in patients with macular edema, particularly if the edema is clinically significant. Recombinant monoclonal neutralizing antibody has been also shown to be of value in patients with diabetic macular edema.

33
Q

Question: 33 of 60

Which of the following can cause a high anion gap metabolic acidosis? (Mark all that are true.)

  1. Severe diarrhea
  2. Ethylene glycol toxicity
  3. Salicylate toxicity
  4. Alcoholic ketoacidosis
  5. Renal tubular acidosis
A
  1. Severe diarrhea
  2. Ethylene glycol toxicity
  3. Salicylate toxicity
  4. Alcoholic ketoacidosis
  5. Renal tubular acidosis

Critique:

A high anion gap metabolic acidosis is defined as a metabolic acidosis associated with an anion gap [serum Na+ – (serum HCO3 + serum Cl)] that exceeds 10–12 mmol/L. The differential diagnosis includes

  • diabetic ketoacidosis,
  • alcoholic ketoacidosis,
  • lactic acidosis,
  • renal failure (acute and chronic),
  • starvation,
  • salicylate toxicity,
  • ethylene glycol toxicity,
  • methanol poisoning, and
  • paraldehyde toxicity.
34
Q

Question: 34 of 60

A 48-year-old overweight female with a 5-year history of type 2 diabetes sees you for a follow-up visit. Her diabetes has been well controlled for the past year with metformin (Glucophage), 850 mg twice daily, and glipizide (Glucotrol XL), 5 mg daily. She is surprised to learn that her hemoglobin A1c has risen to 7.9%, from 6.8% 6 months ago, despite the fact that she has lost 5 lb. In addition to her diabetes medications she takes atorvastatin (Lipitor), 40 mg daily. She began taking vitamin E, 800 units daily, about a year ago. Based on her home glucose monitoring log, her fasting and prandial glucose levels have consistently been below 130 mg/dL and 170 mg/dL, respectively.

Which one of the following is most likely to account for the rise in hemoglobin A1c?

  1. Vitamin E use
  2. Iron deficiency anemia
  3. Hemolytic anemia
  4. Splenomegaly
  5. Glucagonoma
A
  1. Vitamin E use
  2. Iron deficiency anemia
  3. Hemolytic anemia
  4. Splenomegaly
  5. Glucagonoma

Critique:

Hemoglobin A1c (HbA1c) is formed in the blood when glucose attaches to hemoglobin. Since the attachment of glucose to hemoglobin occurs slowly and continuously over the lifespan of erythrocytes, laboratory measurement of HbA1c provides an index of average glucose over the preceding weeks to months. Although the level of HbA1c is affected by all circulating erythrocytes, HbA1c normally represents a weighted average of blood glucose levels during the preceding 120 days (the lifespan of a normal red blood cell). As a result, glucose levels in the preceding 30 days contribute more to the level of HbA1c than do glucose levels 90–120 days earlier.

Any condition that PROLONGS the life of the erythrocyte or is associated with reduced red cell TURNOVER exposes the red cell to glucose for a longer period of time, resulting in higher HbA1c levels. Conditions associated with decreased red blood cell turnover include

  1. iron deficiency anemia,
  2. vitamin B12 deficiency,
  3. folate deficiency, and
  4. asplenia.

Conversely, any condition shortening the lifespan of the erythrocyte or associated with increased red cell turnover would tend to produce lower HbA1c levels. Examples would include hemolytic anemia, splenomegaly, and acute and chronic blood loss. Vitamin E ingestion has also been associated with lower HbA1c levels, presumably by interfering with glycation.

Hemoglobin variants can either falsely raise or lower HbA1c results, depending on the variant and the assay method used. Consequently, the results may be unreliable for diagnosing or monitoring diabetes and prediabetes in patients with hemoglobinopathies. For diabetic patients with an abnormal hemoglobin but normal red cell turnover, such as those with sickle cell trait, an HbA1c assay without interference from abnormal hemoglobins should be used.

Glucagonoma is a rare tumor of the α-cells of the pancreas that results in the overproduction of the hormone glucagon and would raise blood glucose values.

35
Q

Question: 35 of 60

A 67-year-old male with type 2 diabetes mellitus is evaluated for intermittent claudication and is found to have a right ankle-brachial index of 0.65. He has no history of hypertension and his urine is negative for microalbuminuria.

True statements regarding this situation include which of the following? (Mark all that are true.)

  1. Cilostazol (Pletal) has been shown to improve walking distance
  2. Supervised exercise therapy has been shown to improve walking distance
  3. Percutaneous revascularization with iliac artery stenting is as successful in diabetic patients as in nondiabetic patients
  4. Peripheral artery disease is an indication for starting aspirin therapy
A
  1. Cilostazol (Pletal) has been shown to improve walking distance
  2. Supervised exercise therapy has been shown to improve walking distance
  3. Percutaneous revascularization with iliac artery stenting is as successful in diabetic patients as in nondiabetic patients
  4. Peripheral artery disease is an indication for starting aspirin therapy

Critique:

Peripheral vascular disease is classified by the National Cholesterol Education Program as a coronary heart disease equivalent because the associated risk of future major coronary events is comparable to that of established coronary heart disease (>20% over 10 years). Since cardiovascular events remain the principal cause of death in patients with peripheral vascular disease, aspirin therapy should be strongly considered in this population. Two noninvasive interventions that have been shown to improve walking distance in patients with peripheral arterial disease are supervised exercise therapy and the use of cilostazol, a type III phosphodiesterase inhibitor. A randomized, controlled trial found that a home-based walking exercise program improves walking endurance, physical activity, and speed in patients with peripheral artery disease.

Evidence-based guidelines from the American Heart Association and the American College of Cardiology support the use of an ACE inhibitor for secondary prevention in patients with coronary and other vascular diseases (including peripheral artery disease, atherosclerotic aortic disease, and carotid artery disease) who have a left ventricular ejection fraction ≤40%, and in those with hypertension, diabetes mellitus, or chronic kidney disease.

Diabetic patients with peripheral vascular disease tend to have severe occlusive disease below the knee in run-off vessels, which reduces the benefit achievable with percutaneous interventions.

36
Q

Question: 36 of 60

A 45-year-old male with a long history of type 2 diabetes expresses an interest in starting a regular exercise program. Which one of the following complications related to diabetes would preclude him from engaging in a powerlifting program?

  1. Moderate nonproliferative diabetic retinopathy
  2. Cataracts
  3. Peripheral neuropathy
  4. Peripheral artery disease
  5. Diabetic nephropathy
A
  1. Moderate nonproliferative diabetic retinopathy (high risk for vitreous hemorrhage and retinal detachment)
  2. Cataracts
  3. Peripheral neuropathy
  4. Peripheral artery disease
  5. Diabetic nephropathy

Critique:

Although all forms of physical exercise are appropriate for diabetic patients with mild nonproliferative diabetic retinopathy, the American Diabetes Association (ADA) recommends that these patients avoid activities that dramatically elevate blood pressure, such as powerlifting.

Since patients with severe nonproliferative retinopathy and unstable proliferative retinopathy are felt to be at particularly high risk for vitreous hemorrhage and retinal detachment, the ADA recommends that these patients avoid vigorous physical activities of any type, as well as jumping, jarring, and head-down activities; activities that dramatically elevate blood pressure; and breath holding.

No such restrictions are generally recommended for patients with cataracts, peripheral artery disease, peripheral neuropathy, or diabetic kidney disease (SOR C).

37
Q

Question: 37 of 60

A 55-year-old male with type 2 diabetes mellitus has a chronic history of reduced libido and erectile dysfunction. On examination you note hepatomegaly and mild testicular atrophy. You perform a nonfasting laboratory workup, with the following serum levels reported:

Glucose…………250 mg/dL
AST (SGOT)…………260 U/L (N 10–40)
ALT (SGPT)…………210 U/L (N 10–55)
FSH…………5.0 U/mL (N 1.0–12.0)
LH…………8.1 U/mL (N 2.0–12.0)
Testosterone…………180 ng/mL (N 280–1250)

What is the most likely diagnosis?

  1. Glucagonoma
  2. Hemochromatosis
  3. Pheochromocytoma
  4. Acromegaly
  5. Cushing’s syndrome
A
  1. Glucagonoma
  2. Hemochromatosis
  3. Pheochromocytoma
  4. Acromegaly
  5. Cushing’s syndrome

Critique:

Hemochromatosis results from excessive iron storage. Its idiopathic form, seen primarily in European populations, is one of the most common autosomal recessive disorders. It arises from the massive deposition of iron in parenchymal tissues (especially the liver, pancreas, pituitary gland, and heart), causing fibrotic changes and functional impairment of the involved organs. Symptoms usually develop between the ages of 40 and 60 years. Classic clinical features include bronze skin pigmentation, diabetes mellitus, hepatomegaly with hepatic dysfunction, cardiac failure, and evidence of hypogonadism. Screening and greater awareness of the disease among clinicians has facilitated recognition of the disease at earlier stages; common symptoms now include fatigue, malaise, arthralgia, and hepatomegaly.

38
Q

Question: 38 of 60

A 72-year-old male sees you for a routine follow-up visit. His past medical history is notable for type 2 diabetes, hyperlipidemia, stage 3 chronic kidney disease, TIAs, carotid endarterectomy, and bladder cancer. His current medications are metformin (Glucophage), 500 mg twice daily; sitagliptin (Januvia), 50 mg/day; nateglinide (Starlix), 120 mg 3 times daily with meals; simvastatin, (Zocor), 40 mg/day; and aspirin, 81 mg/day.

The patient’s blood pressure is 134/76 mm Hg and his BMI is 28 kg/m2. The physical examination is notable only for scattered actinic keratoses on his forearms and absent pedal pulses.

Laboratory Findings

Serum creatinine…………1.9 mg/dL (N 0.6–1.5)
Estimated glomerular filtration rate…………52 mL/min/1.73 m2
BUN…………45 mg/dL (N 8-25)
Hemoglobin A1c…………7.8 %
LDL-cholesterol…………95 mg/dL
HDL-cholesterol…………33 mg/dL
Serum triglycerides…………163 mg/dL

Which of the following would be appropriate at this time? (Mark all that are true.)

  1. Increasing the dosage of simvastatin to 80 mg/day
  2. Starting basal insulin
  3. Starting extended-release niacin, 500 mg/day
  4. Starting lisinopril (Prinivil, Zestril), 5 mg/day
  5. Starting pioglitazone (Actos), 15 mg/day
A
  1. Increasing the dosage of simvastatin to 80 mg/day
  2. Starting basal insulin (no need – his HA1C < 8.0)
  3. Starting extended-release niacin, 500 mg/day
  4. Starting lisinopril (Prinivil, Zestril), 5 mg/day
  5. Starting pioglitazone (Actos), 15 mg/day
39
Q

Question: 39 of 60

A 60-year-old groundskeeper is brought to the emergency department unconscious. His temperature is 38.1°C (100.6°F) rectally, blood pressure 96/70 mm Hg, pulse 128 beats/min, and respirations 15/min. The examination is otherwise unremarkable except for very dry skin and mucous membranes.

Laboratory Findings

Serum sodium…………150 mEq/L (N 135–145)
Serum potassium…………3.1 mEq/L (N 3.5–5.0)
Serum chloride…………112 mEq/L (N 100–108)
CO2…………26 mEq/L (N 24–30)
Serum glucose…………1080 mg/dL
Serum creatinine…………4.0 mg/dL (N 0.6–1.5)
BUN…………70 mg/dL (N 8–25)
Serum ketones…………small amount present

Which one of the following is the most likely diagnosis?

  1. Diabetic ketoacidosis
  2. Diabetes mellitus with lactic acidosis
  3. Diabetes mellitus with sepsis
  4. Hyperosmolar, hyperglycemic state
  5. Paraldehyde toxicity
A
  1. Diabetic ketoacidosis
  2. Diabetes mellitus with lactic acidosis
  3. Diabetes mellitus with sepsis
  4. Hyperosmolar, hyperglycemic state
  5. Paraldehyde toxicity

Critique:

The term hyperosmolar, hyperglycemic state has replaced the older terms hyperglycemic, hyperosmolar, nonketotic coma and hyperglycemic, hyperosmolar, nonketotic state. Criteria for the diagnosis include the following:

  • a plasma glucose level >600 mg/dL
  • an arterial pH >7.30
  • a serum bicarbonate level >15 mEq/L
  • effective serum osmolality >320 mOsm/kg
  • variable anion gap >12
  • no more than small serum or urine ketones
  • a stuporous or comatose state
40
Q

Question: 40 of 60

A 71-year-old male with a history of type 2 diabetes mellitus and long-standing hypertension sees you because of worsening ankle edema, weight gain, and “getting more winded” when climbing stairs. His current medications are glipizide (Glucotrol), 10 mg/day; pioglitazone (Actos), 30 mg/day; extended-release metformin (Glucophage XR), 1000 mg/day; acarbose (Precose), 25 mg three times a day; lisinopril (Prinivil, Zestril), 40 mg/day; and hydrochlorothiazide, 12.5 mg/day.

Which one of his medications is most likely responsible for his symptoms?

  1. Metformin
  2. Glipizide
  3. Pioglitazone
  4. Acarbose
A
  1. Metformin
  2. Glipizide
  3. Pioglitazone
  4. Acarbose

Critique:

Weight gain and edema occur in 3%–5% of patients treated with thiazolidinediones. Because of their propensity to expand plasma volume, the use of these agents is contraindicated in patients with class III or IV heart failure. Two meta-analyses have shown an increased risk of heart failure in patients treated with pioglitazone and rosiglitazone. Following its review of post-marketing adverse event reports, the FDA announced on August 14, 2007, that the manufacturers of both of these agents agreed to add a stronger warning to the package insert about the risk of their products causing or worsening heart failure.

41
Q

Question: 41 of 60

A 58-year-old male is diagnosed with type 2 diabetes. His past medical history is notable for a long history of hypertension, stage 3 chronic kidney disease, low back pain, and osteoarthritis of his knees. His current medications are amlodipine (Norvasc), 10 mg daily; valsartan (Diovan), 180 mg daily; chlorthalidone, 30 mg daily; and simvastatin (Zocor), 40 mg daily. Laboratory findings are notable only for a hemoglobin A1c of 7.9%, a serum creatinine level of 1.6 mg/dL (N 0.7–1.3), and an estimated glomerular filtration rate of 53 mL/min/1.73 m2.

Which one of the following diabetes agents would be unlikely to improve this patient’s glycemic control?

  1. Dapagliflozin (Farxiga)
  2. Glipizide (Glucotrol)
  3. Liraglutide (Victoza)
  4. Metformin (Glucophage)
  5. Sitagliptin (Januvia)
A
  1. Dapagliflozin (Farxiga)
  2. Glipizide (Glucotrol)
  3. Liraglutide (Victoza)
  4. Metformin (Glucophage)
  5. Sitagliptin (Januvia)

Critique:

SGLT2 inhibitors (Canagli-flozin, Dapagli-flozin) improve glucose control by preventing reabsorption of glucose and increasing urinary glucose excretion.

  • Consequently, their ability to lower glucose is directly proportional to the glomerular filtration rate (“FLO”) and is therefore reduced in patients with chronic kidney disease (CKD).
  • In patients with an estimated glomerular filtration rate (eGFR) <60 mL/min/1.73 m2 glycemic control is not improved by dapagliflozin compared with placebo.
  • The same is true for patients with an eGFR <45 mL/min/1.73 m2 who take empagliflozin or canagliflozin.

Liraglutide and glipizide can be safely used in patients with CKD. The kidneys are not a major pathway of elimination for liraglutide or glipizide and dose adjustments are generally not necessary in patients with CKD. The dipeptidyl peptidase-4 (DPP-4) inhibitors are useful agents in diabetic patients with CKD, although lowering dosages based on the eGFR is required for all of them except linagliptin. The use of metformin was contraindicated in the past for males with serum creatinine levels ≥1.5 mg/dL and females with serum creatinine levels ≥1.4 mg/dL, but in 2016 the FDA issued a Drug Safety Communication that stated that the use of metformin is contraindicated in individuals with an eGFR <30 mL/min/1.73 m2 and advised clinicians to start metformin only in patients with an eGFR >45 mL/min/1.73 m2 and to obtain an eGFR at least annually. The FDA also advised that if or when the eGFR falls below 45 mL/min/1.73 m2 metformin should be continued only after assessment of the benefits and risks, and that it be discontinued if the eGFR fell below 30 mL/min/1.73 m2.

42
Q

Question: 42 of 60

Which one of the following agents is most likely to produce weight loss in the diabetic patient?

  1. Thiazolidinediones
  2. GLP-1–receptor agonists
  3. Sulfonylureas
  4. Metformin (Glucophage)
  5. α-Glucosidase inhibitors
A
  1. Thiazolidinediones
  2. GLP-1–receptor agonists (exenatide and liraglutide)
  3. Sulfonylureas
  4. Metformin (Glucophage)
  5. α-Glucosidase inhibitors

Critique:

Of all available agents used for the treatment of type 2 diabetes mellitus, the GLP-1–receptor agonists (exenatide and liraglutide) are most likely to result in weight reduction. Metformin is generally considered to be weight neutral with chronic use. Weight loss is not typically seen in patients treated with an α-glucosidase inhibitor. Weight gain is commonly seen in patients treated with insulin, sulfonylureas, meglitinides, and thiazolidinediones.

43
Q

Question: 43 of 60

What is the minimum degree of weight loss recommended to reduce the risk of diabetes mellitus in a patient with impaired glucose tolerance?

  1. Weight reduction of 2%–4%
  2. Weight reduction of 5%–10%
  3. Weight reduction of 10%–20%
  4. Weight reduction of 20%–30%
  5. Achievement of ideal body weight
A
  1. Weight reduction of 2%–4%
  2. Weight reduction of 5%–10%
  3. Weight reduction of 10%–20%
  4. Weight reduction of 20%–30%
  5. Achievement of ideal body weight

Critique:

In a Finnish study of 522 middle-aged men with impaired glucose tolerance, intensive individualized instruction regarding weight loss, food intake, and physical activity resulted in a 58% reduction in the incidence of diabetes mellitus compared to the control group. In this study, weight reduction as small as 5%–10% was found to lower the risk of developing diabetes mellitus. Similarly, the Diabetes Prevention Program Research Group found a reduction in the incidence of diabetes mellitus in persons at high risk using a lifestyle modification program with goals of at least a 7% weight loss and at least 150 minutes of physical activity per week.

44
Q

Question: 44 of 60

A 39-year-old female with type 2 diabetes mellitus develops microalbuminuria and is started on enalapril (Vasotec). At a follow-up visit 2 months later, an electrolyte panel reveals an unchanged serum creatinine level of 1.4 mg/dL, but her potassium level has risen from a baseline of 4.0 mEq/L to its present level of 5.4 mEq/L (N 3.5–5.0).

Which one of the following is the most likely cause of her potassium elevation?

  1. Diabetic glomerulosclerosis
  2. Hyporeninemic hypoaldosteronism
  3. Hyperaldosteronism
  4. Hemolytic anemia
  5. Bilateral renal artery stenosis
A
  1. Diabetic glomerulosclerosis
  2. Hyporeninemic hypoaldosteronism
  3. Hyperaldosteronism
  4. Hemolytic anemia
  5. Bilateral renal artery stenosis

Critique:

The use of ACE inhibitors or ARBs can exacerbate hyperkalemia in patients with advanced renal disease and/or HYPOreninemic HYPOaldosteronism.

ACE inhibitors can cause a rapid decline in renal function in older patients with bilateral renal artery stenosis and in patients with advanced renal disease.

45
Q

Question: 45 of 60

A 42-year-old female with a BMI of 31.0 kg/m2 has a 3-week history of polyuria and polydipsia, accompanied by a 10-lb weight loss. Her fasting plasma glucose level is 320 mg/dL, and her hemoglobin A1c is 11.1%.

Which one of the following is most likely to reverse her glucose toxicity and improve her glycemic response?

  1. Metformin (Glucophage)
  2. Pioglitazone (Actos)
  3. Glipizide (Glucotrol)
  4. Acarbose (Precose)
  5. Insulin
A
  1. Metformin (Glucophage)
  2. Pioglitazone (Actos)
  3. Glipizide (Glucotrol)
  4. Acarbose (Precose)
  5. Insulin

Critique:

High levels of glucose are toxic to pancreatic β-cells, leading to impairment of insulin secretion. Initial treatment with insulin not only allows for more rapid glycemic control, but also facilitates more rapid recovery of β-cell function. This in turn fosters an improved subsequent response to oral agents.

46
Q

Question: 46 of 60

A 60-year-old female sees you for her annual checkup. Her past medical history is notable for a 15-year history of type 2 diabetes and hypercholesterolemia. Her current medications are extended-release metformin (Glucophage XR), 2000 mg/day; extended-release glipizide (Glucotrol XL), 5 mg/day; atorvastatin (Lipitor), 10 mg/day; and aspirin, 81 mg/day.

The physical examination is unremarkable. The patient’s blood pressure is 128/78 mm Hg and her BMI is 29.1 kg/m2. Laboratory testing reveals a hemoglobin A1C of 7.2%, an LDL-cholesterol level of 95 mg/dL, an HDL-cholesterol level of 36 mg/dL, and a serum triglyceride level of 190 mg/dL.

The patient tells you that she plans to start “jogging,” and you order an exercise nuclear stress test which reveals findings suspicious for exercise-induced ischemia. Coronary angiography reveals a 65% stenosis of the mid-right coronary artery.

True statements regarding this situation include which of the following? (Mark all that are true.)

  1. The patient’s aspirin dosage should be increased to 325 mg/day since it is now for secondary prevention
  2. The patient’s atorvastatin dosage should be increased
  3. The patient’s glipizide dosage should be increased
  4. Prompt revascularization has been shown to be superior to intensive medical therapy in terms of mortality and major cardiovascular events
  5. Percutaneous coronary intervention and coronary artery bypass graft (CABG) surgery are equally effective in patients with diabetes mellitus and coronary heart disease
A
  1. The patient’s aspirin dosage should be increased to 325 mg/day since it is now for secondary prevention
  2. The patient’s atorvastatin dosage should be increased
  3. The patient’s glipizide dosage should be increased
  4. Prompt revascularization has been shown to be superior to intensive medical therapy in terms of mortality and major cardiovascular events
  5. Percutaneous coronary intervention and coronary artery bypass graft (CABG) surgery are equally effective in patients with diabetes mellitus and coronary heart disease

Critique:

In 2018, the American Diabetes Association (ADA) recommended that patients of all ages with diabetes and atherosclerotic cardiovascular disease (ASCVD) should have high-intensity statin therapy added to lifestyle therapy (SOR A).

In addition, the ADA recommended that patients with diabetes and an LDL-C 70 mg/dL who are 40–75 years of age (SOR A) or >75 years of age (SOR B) who do not have ASCVD should have moderate-intensity statin therapy in addition to lifestyle therapy. This is consistent with the 2018 guidelines from the American Heart Association (AHA), which recommend that high-intensity statin therapy be initiated or continued in patients who are 75 years of age or younger with clinical ASCVD (SOR A) and moderate-intensity statin therapy be prescribed in adults 40–75 years of age with diabetes mellitus, regardless of their estimated 10-year ASCVD risk (SOE A).

Aspirin at a dosage of 75–162 mg/day is recommended as a secondary preventive strategy in patients with diabetes and a history of cardiovascular disease, as well as a primary prevention strategy for those with type 1 or type 2 diabetes who are at increased cardiovascular risk. This includes most men and women with diabetes aged >50 years who have at least one additional major risk factor (family history of premature ASCVD, hypertension, dyslipidemia, smoking, or albuminuria) and are not at increased risk of bleeding.

In light of this patient’s age, duration of diabetes, and known cardiovascular disease, a hemoglobin A1C target of around 7% would be most appropriate.

For patients with diabetes mellitus and coronary heart disease, revascularization with coronary artery bypass graft (CABG) surgery has been shown to be superior to percutaneous coronary intervention in terms of survival and/or other clinical outcomes. This was shown in the FREEDOM trial (Future Revascularization Evaluation in Patients with Diabetes Mellitus: Optimal Management of Multivessel Disease), a randomized trial of 1900 patients with diabetes and multivessel coronary heart disease. Treatment with CABG was associated with both a lower rate of myocardial infarction and lower mortality compared to PCI with drug-eluting stents.

The Bypass Angioplasty Revascularization Investigation 2 Diabetes (BARI 2D trial) of 2368 patients with both diabetes and coronary heart disease found NO significant difference in the rates of death and major cardiovascular events between patients undergoing prompt revascularization and those provided optimal medical treatment.

47
Q

Question: 47 of 60

Patients must eat within 15 minutes of administration of which one of the following types of insulin?

  1. Lente
  2. Ultralente
  3. Insulin glargine
  4. NPH
  5. Insulin lispro
A
  1. Lente
  2. Ultralente
  3. Insulin glargine
  4. NPH
  5. Insulin lispro

Critique:

Insulin lispro is a rapid-acting insulin that should be injected within 15 minutes before a meal or immediately after a meal to avoid hypoglycemia.

NPH and lente are intermediate-acting insulin types with an onset of action of 1–3 hours.

Ultralente and insulin glargine are long-acting types with an onset of action of 2–4 hours and 1–2 hours, respectively.

48
Q

Question: 48 of 60

A 61-year-old male with a history of type 2 diabetes, hypertension, and vascular claudication sees you for a follow-up visit. His diagnosis of vascular claudication was based on a right ankle-brachial index of 0.85. His current medications are ramipril (Altace), 10 mg daily; metformin (Glucophage), 850 mg twice daily; rosuvastatin (Crestor), 10 mg daily; and aspirin, 81 mg daily. A physical examination is notable for a callus on his right first toe. Other findings include absent sensation on 10-g monofilament testing and reduced vibration sensation using a 128-Hz tuning fork placed on the interphalangeal joint of the great toe. Laboratory testing is notable only for a hemoglobin A1c of 8.1%.

Which one of the following diabetes agents should be avoided because it is associated with a higher risk for amputation?

  1. Basal insulin
  2. Canagliflozin (Invokana)
  3. Liraglutide (Victoza)
  4. Pioglitazone (Actos)
  5. Sitagliptin (Januvia)
A
  1. Basal insulin
  2. Canagliflozin (Invokana) (SGLT-1 inhibitor)
  3. Liraglutide (Victoza)
  4. Pioglitazone (Actos)
  5. Sitagliptin (Januvia)

Critique:

The FDA has issued a Drug Safety Communication warning clinicians of a higher risk of amputation reported in patients with diabetes treated with canagliflozin.

The warning was based on findings from the CANVAS (Canagliflozin Cardiovascular Assessment Study) and CANVAS-R (A Study of the Effects of Canagliflozin on Renal Endpoints in Adult Participants With Type 2 Diabetes Mellitus) trials, both of which demonstrated that leg and foot amputations occurred about twice as often in patients taking canagliflozin.

Although evidence of a higher risk of amputations has not been reported for other SGLT2 inhibitors, the possibility of a class effect has not been excluded. The FDA advises that before initiating canagliflozin, clinicians should consider factors in the patient’s history that may predispose them to the need for amputations, such as a history of prior amputation, peripheral vascular disease, neuropathy, or diabetic foot ulcers.

Patients receiving canagliflozin should be monitored for signs and symptoms of infection, new pain or tenderness, sores, or ulcers involving the lower limbs, and canagliflozin should be discontinued if these complications occur (SOR C).

49
Q

Question: 49 of 60

You are evaluating a patient with diabetes mellitus and hypertension with 24-hour ambulatory blood pressure monitoring. You note a rise in systolic blood pressure during sleep.

This has been shown to be an early indicator of which one of the following?

  1. Microalbuminuria
  2. Orthostatic hypotension
  3. Gustatory sweating
  4. Proliferative diabetic retinopathy
  5. Systolic hypertension
A
  1. Microalbuminuria
  2. Orthostatic hypotension
  3. Gustatory sweating
  4. Proliferative diabetic retinopathy
  5. Systolic hypertension

Critique:

A circadian reduction in blood pressure normally occurs during sleep, with the ratio of nighttime to daytime blood pressure being 0.9 or lower. In patients with type 1 diabetes mellitus, the development of a paradoxical rise in systolic blood pressure during sleep appears to predict the development of microalbuminuria and incipient diabetic nephropathy. A similar finding has been demonstrated in patients with type 2 diabetes as well.

50
Q

Question: 50 of 60

The strongest predictor for the development and progression of diabetic retinopathy is

  1. glycemic control
  2. blood pressure
  3. lipid levels
  4. duration of disease
  5. smoking history
A
  1. glycemic control
  2. blood pressure
  3. lipid levels
  4. duration of disease
  5. smoking history

Critique: Time is the most important risk factor for the development of diabetic retinopathy. In type 1 diabetics, retinopathy is uncommon in the first 5 years after disease onset, but occurs in 25%–50% of patients after 10–15 years and in 95% of patients after 30 years. Among patients with type 2 diabetes, approximately 10% will have nonproliferative retinal disease at 10 years, 40% at 15 years, and 60% at 20 years.

51
Q

Question: 51 of 60

The most common cause of sudden monocular loss of vision in a patient with diabetic retinopathy is

  1. acute glaucoma
  2. vertebrobasilar stroke
  3. central retinal vein occlusion
  4. ischemic optic neuropathy
  5. vitreous hemorrhage
A
  1. acute glaucoma
  2. vertebrobasilar stroke
  3. central retinal vein occlusion
  4. ischemic optic neuropathy
  5. vitreous hemorrhage

Critique:

Sudden loss of vision can occur for a number of reasons in patients with diabetes mellitus, the most common being vitreous hemorrhage.

Less common causes that should be considered include vascular occlusion (central retinal vein or branch vein occlusion involving the macula), retinal detachment, end-stage glaucoma, and ischemic optic neuropathy.

52
Q

Question: 52 of 60

A 68-year-old female with longstanding type 2 diabetes mellitus and hypertension comes in for her annual examination. Her current medications are metformin (Glucophage), 850 mg twice daily; glipizide (Glucotrol), 10 mg daily; atorvastatin (Lipitor) 40 mg daily; and lisinopril (Prinivil, Zestril), 20 mg daily.

The American Diabetes Association recommends that this patient be periodically assessed for which one of the following vitamin deficiencies?

  1. Folate
  2. Thiamine
  3. Vitamin B12
  4. Vitamin C
  5. Vitamin D
A
  1. Folate
  2. Thiamine
  3. Vitamin B12
  4. Vitamin C
  5. Vitamin D

Critique:

Metformin is the most frequently prescribed first-line therapy for individuals with type 2 diabetes. It has been shown to reduce intestinal absorption of vitamin B12, and the risk of vitamin B12 deficiency in patients treated with metformin has been correlated with the dosage and duration of use.

A secondary analysis of patients from the Diabetes Prevention Program Outcomes Study found that long-term use of metformin was associated with biochemical vitamin B12 deficiency and anemia.

The American Diabetes Association recommends periodic measurement of vitamin B12 in patients treated with metformin, especially those with anemia or neuropathy (SOR B).

53
Q

Question: 53 of 60

A 16-year-old male has a 1-week history of polyuria, polydipsia, and polyphagia. On laboratory evaluation he is found to have a serum glucose level of 270 mg/dL, a serum bicarbonate level of 9 mEq/L (N 22–26), a serum pH of 7.0, and a serum potassium level of 4.0 mEq/L (N 3.5–5.0).

Which one of the following most accurately describes this patient’s total body potassium?

  1. Mild total body potassium excess
  2. Normal total body potassium stores
  3. Mild total body potassium deficiency
  4. Severe total body potassium deficiency
A
  1. Mild total body potassium excess
  2. Normal total body potassium stores
  3. Mild total body potassium deficiency
  4. Severe total body potassium deficiency

Critique:

Excessive urinary potassium losses stemming from the osmotic diuresis associated with a hyperglycemic crisis can result in severe total body potassium deficits of 500–700 mmol. Although the finding of a “normal” serum potassium may appear reassuring, it is misleading since extracellular hyperosmolarity and the metabolic acidosis seen in diabetic ketoacidosis combine to cause a shift of potassium and water from the intracellular to the extracellular space. This shift can raise serum potassium levels to normal or even elevated levels in the face of a severe potassium deficit. The measured serum potassium increases approximately 0.7 mmol/L for each 0.1-unit decrease in pH. In this case, the pH has declined 0.4 from a normal of 7.4, which would create an approximate 2.8-mmol/L increase in serum potassium. The measured potassium of 4.0 mmol/L would thus correspond to an actual serum potassium of 1.2 mmol/L. A decline of 1.0 mmol/L in serum potassium concentration corresponds to a 200–400 mmol deficit. In this case, it amounts to well over 400 mmol.

54
Q

Question: 54 of 60

A 58-year-old male with type 2 diabetes mellitus has a blood pressure of 147/92 mm Hg. You start him on benazepril (Lotensin) and order a baseline serum creatinine level, which is 1.7 mg/dL. Two weeks later his blood pressure is 128/80 mm Hg, and his serum creatinine level is 2.1 mg/dL. His creatinine level is unchanged 1 week later.

Which one of the following is the most appropriate course of action?

  1. Continue benazepril at the same dosage
  2. Reduce the benazepril dosage
  3. Discontinue benazepril
  4. Evaluate the patient for bilateral renal artery stenosis
  5. Have the patient increase his sodium intake
A
  1. Continue benazepril at the same dosage
  2. Reduce the benazepril dosage
  3. Discontinue benazepril
  4. Evaluate the patient for bilateral renal artery stenosis
  5. Have the patient increase his sodium intake

Critique:

An initial decline in renal function is not uncommon in the hypertensive patient whose blood pressure is brought under control. This decline is generally considered to be functional and is associated with long-term renal protection. In the patient who experiences good blood pressure control with antihypertensive therapy, stabilization of serum creatinine levels after an initial 20%–30% rise indicates that the intraglomerular pressure has been successfully reduced. ACE inhibitors should be discontinued if the creatinine level increases by 30% above baseline in the first 2 months of therapy.

55
Q

Question: 55 of 60

Mechanisms of action of exenatide (Byetta) include which of the following? (Mark all that are true.)

  1. Enhanced insulin secretion
  2. Suppression of glucagon secretion
  3. Enhanced insulin sensitivity of muscle
  4. Slowing of gastric motility
  5. Reduction of the rate of polysaccharide digestion in the small intestine
A
  1. Enhanced insulin secretion
  2. Suppression of glucagon secretion
  3. Enhanced insulin sensitivity of muscle
  4. Slowing of gastric motility
  5. Reduction of the rate of polysaccharide digestion in the small intestine

Critique:

Incretin hormones, which include glucagon-like peptide 1 (GLP-1) and glucose-independent insulinotropic polypeptide (GIP), are released from the gastrointestinal tract after a meal.

These hormones are responsible for 70% of postprandial insulin secretion.

Exenatide is a synthetic GLP-1 that is administered subcutaneously twice a day and potentiates glucose-mediated insulin secretion.

  • In addition, exenatide may lower blood glucose levels by suppressing glucagon secretion and slowing gastric motility. By lowering postprandial blood glucose levels, exenatide lowers hemoglobin A1c by 0.5–1.0 percentage points.
56
Q

Question: 56 of 60

A 51-year-old female sees you for her annual examination. Her past medical history is notable only for a 2-year history of type 2 diabetes mellitus. Her only medication is metformin (Glucophage), 500 mg twice daily before breakfast and dinner. She is a nonsmoker. Her family history is notable for her mother having a myocardial infarction in her late 50s. A physical examination is unremarkable, and her blood pressure is 128/76 mm Hg. Laboratory evaluation reveals a serum creatinine level of 0.8 mg/dL (N 0.6–1.5), a hemoglobin A1c of 6.9%, and no microalbuminuria. Her lipid profile includes an LDL-cholesterol level of 105 mg/dL, an HDL-cholesterol level of 42 mg/dL, and a serum triglyceride level of 160 mg/dL. She has worked diligently on lifestyle therapy with a dietician.

According to current American Diabetes Association guidelines, which of the following interventions would be appropriate? (Mark all that are true.)

  1. Beginning an ACE inhibitor or angiotensin receptor blocker
  2. Beginning aspirin, 81 mg/day
  3. Beginning a moderate-intensity statin
  4. Beginning a fibric acid derivative
A
  1. Beginning an ACE inhibitor or angiotensin receptor blocker
  2. Beginning aspirin, 81 mg/day
  3. Beginning a moderate-intensity statin
  4. Beginning a fibric acid derivative

Critique:

In its 2020 guidelines, the American Diabetes Association (ADA) recommends that patients 40—75 years of age with diabetes mellitus and without atherosclerotic cardiovascular disease use moderate-intensity statin therapy in addition to lifestyle therapy (level A recommendation). The 2020 guidelines also state that combination therapy with a statin plus a fibrate has not been shown to improve atherosclerotic cardiovascular disease outcomes and is generally not recommended (level A recommendation). Combination therapy with a statin plus niacin is not recommended because it has not been shown to provide additional cardiovascular benefit above statin therapy alone, and it may increase the risk of stroke (level A recommendation).

As a new recommendation, the 2020 guidelines note that patients with atherosclerotic cardiovascular disease or other cardiovascular risk factors on a statin with controlled LDL-cholesterol levels but continued elevated triglycerides (135–499 mg/dL), the addition of icosapent ethyl can be considered to reduce cardiovascular risk (level A recommendation). This additional medication could be considered for this patient if her triglyceride level is still elevated after the statin therapy.

For many nonpregnant adults, a hemoglobin A1c goal of <7% is appropriate. The ADA guidelines state that an ACE inhibitor or angiotensin receptor blocker is not recommended for the primary prevention of chronic kidney disease in patients with diabetes who have normal blood pressures, normal urine albumin/creatinine ratios (<30 mg/g), and normal estimated glomerular filtration rates.

Aspirin, 75–162 mg daily, may be considered a primary prevention strategy in those with diabetes who are at increased cardiovascular risk, after a comprehensive discussion with the patient about the benefits and the increased risk of bleeding (level A recommendation). In primary prevention, however, among patients with no previous cardiovascular events, the net benefit of aspirin therapy is more controversial. Recommendations for using aspirin as primary prevention include both men and women aged ≥50 years with diabetes and at least one additional major risk factor, including hypertension, dyslipidemia, smoking, chronic kidney disease or albuminuria, or a family history of premature atherosclerotic cardiovascular disease, who are not at increased risk of bleeding due to factors such as older age, anemia, or renal disease. The main adverse effect is an increased risk of gastrointestinal bleeding.

57
Q

Question: 57 of 60

A 58-year-old obese male comes to your office with a 2-week history of fatigue associated with polyuria, polydipsia, and weight loss. You suspect he has type 2 diabetes mellitus. This diagnosis would be corroborated by a random glucose level greater than or equal to _________ mg/dL

A

200 mg/dL

Critique:

Current American Diabetes Association guidelines specify that the diagnosis of diabetes mellitus should be made in a patient with symptoms of diabetes (e.g., polyuria, polydipsia, unexplained weight loss) and a random plasma glucose concentration of 200 mg/dL or more.

58
Q

Question: 58 of 60

A 64-year-old female accountant with a 6-year history of type 2 diabetes presents with a 2-day history of malaise, dizziness, nausea, and vomiting. Her temperature is 37.8°C (100.0°F), blood pressure 96/70 mm Hg, pulse rate 108 beats/min, and respiratory rate 20/min. The examination is otherwise unremarkable except for a BMI of 29.2 kg/m2 and dry mucous membranes.

Laboratory Findings

Serum sodium…………135 mEq/L (N 135–145)
Serum potassium…………3.9 mEq/L (N 3.5–5.0)
Serum chloride…………103 mEq/L (N 100–108)
CO2…………15 mEq/L (N 24–30)
Serum glucose…………224 mg/dL
Serum creatinine…………0.67 mg/dL (N 0.6–1.5)
BUN…………20 mg/dL (N 8–25)
Serum ketones…………small amount present
Arterial pH…………7.12 (N 7.35–7.45)
Hemoglobin A1c…………7.4%

Which one of the following diabetes agents is most likely to be associated with this presentation?

  1. Canagliflozin (Invokana)
  2. Exenatide (Byetta)
  3. Glipizide (Glucotrol)
  4. Linagliptin (Tradjenta)
  5. Nateglinide (Starlix)
A
  1. Canagliflozin (Invokana)
  2. Exenatide (Byetta)
  3. Glipizide (Glucotrol)
  4. Linagliptin (Tradjenta)
  5. Nateglinide (Starlix)

Critique:

In May 2015, the FDA issued a Drug Safety Communication that warned of a higher risk of diabetic ketoacidosis (DKA) with uncharacteristically mild or moderate glucose elevations associated with use of all available SGLT2 inhibitors.

Actions associated with these agents that might contribute to a higher risk for euglycemic diabetic ketoacidosis include increases in urine glucose excretion, development of hyperglucagonemia, and the potential for volume depletion. Although the majority of cases have been reported in patients with type 2 diabetes, a few cases have been seen in patients with type 1 diabetes as well. Intercurrent illness, reduced food and fluid intake, reduced insulin doses, and a history of alcohol intake have been reported as possible contributing factors. The possibility of euglycemic diabetic ketoacidosis warrants consideration in all diabetic patients taking an SGLT2 inhibitor who develop nausea, vomiting, and malaise. Patients found to have ketonemia or ketonuria should be evaluated for the presence of metabolic acidosis even if glucose levels appear nearly normal. Diagnostic criteria for DKA includes an arterial pH <7.3, positive serum ketones, an anion gap >10, and elevated β-hydroxybutyrate levels. Management of euglycemic ketoacidosis involves discontinuation of the SGLT2 inhibitor and initiation of a DKA protocol that includes a rapid-acting insulin with carbohydrate-containing fluids.

59
Q

Question: 59 of 60

A 16-year-old female is admitted to the hospital with a 1-week history of polyuria, polydipsia, and polyphagia. Examination reveals a lethargic, volume-depleted female with the smell of acetone on her breath. Her blood pressure is 96/70 mm Hg, her pulse rate is 120 beats/min, and she has Kussmaul respirations at a rate of 32/min.

Laboratory Findings

Serum glucose…………525 mg/dL
Serum sodium…………122 mEq/L (N 135–145)
Serum potassium…………3.1 mEq/L (N 3.5–5.0)
Serum chloride…………95 mEq/L (N 100–108)
CO2…………7 mEq/L (N 24–30)
Arterial blood gases
pH…………7.10 (N 7.35–7.45)
pCO2…………15 mm Hg (N 35–45)
pO2…………98 mm Hg (N 80–100)

After initiation of intravenous fluid therapy, which one of the following should be done next?

  1. Initiation of insulin therapy
  2. Potassium replacement
  3. Bicarbonate therapy
  4. Phosphate therapy
  5. Dexamethasone therapy
A
  1. Initiation of insulin therapy
  2. Potassium replacement
  3. Bicarbonate therapy
  4. Phosphate therapy
  5. Dexamethasone therapy

Critique:

The therapeutic goals for the treatment of a hyperglycemic crisis consist of: (1) improving circulatory volume and tissue perfusion; (2) reducing plasma glucose and plasma osmolality to normal levels; (3) clearing the serum and urine of ketones at a steady rate; (4) correcting electrolyte imbalances; and (5) identifying and treating precipitating events. Restoration of fluid and electrolyte deficits is the first priority when treating patients with hyperglycemic crisis. Patients should initially be given 1 L of 0.9% NaCl solution intravenously over the first hour, followed by either 0.9% NaCl or 0.45% NaCl solution as dictated by the patient’s corrected serum sodium and hemodynamic status. Restoration of intravascular volume is important, since it serves to decrease counterregulatory hormones, lower blood glucose, and augment insulin sensitivity.

Regular insulin is the next intervention usually made (while the patient is being hydrated with the first liter of normal saline) unless significant hypokalemia (serum potassium <3.3 mEq/L) is present. In the presence of severe hypokalemia, potassium replacement needs to be started before the initiation of insulin therapy, since the latter would produce a rapid decline in the already-low plasma potassium level, due to insulin-mediated reentry of potassium to the intracellular compartment. Insulin therapy should be withheld until plasma potassium levels exceed 3.3 mEq/L. If insulin is given, it is initially administered as a 0.1-U/kg intravenous bolus, followed by an intravenous insulin infusion at a rate of 0.1 U/kg/hr. Alternatively, it may be given as a continuous infusion at a rate of 0.14 U/kg/hr without an initial bolus.

60
Q

Question: 60 of 60

According to the current American Diabetes Association guidelines, statin therapy should be considered for which of the following patients with diabetes mellitus? (Mark all that apply.)

  1. A 36-year-old male nonsmoker with normal blood pressure, a normal BMI, an LDL-cholesterol level of 95 mg/dL, an HDL-cholesterol level of 50 mg/dL, and a triglyceride level of 195 mg/dL
  2. A 45-year-old female nonsmoker with a normal blood pressure and BMI, an LDL-cholesterol level of 120 mg/dL, an HDL-cholesterol level of 56 mg/dL, and a triglyceride level of 105 mg/dL
  3. A 35-year-old obese male smoker with prehypertension, an LDL-cholesterol level of 125 mg/dL, an HDL-cholesterol level of 39 mg/dL, and a triglyceride level of 125 mg/dL
  4. A 72-year-old normotensive male nonsmoker with a BMI of 23.1 kg/m2, a past history of stroke, an LDL-cholesterol level of 65 mg/dL, an HDL-cholesterol level of 50 mg/dL, and a triglyceride level of 145 mg/dL
  5. A 65-year-old female nonsmoker with a normal blood pressure and BMI, an LDL-cholesterol level of 80 mg/dL, an HDL-cholesterol level of 50 mg/dL, and a triglyceride level of 294 mg/dL
A
  1. A 36-year-old male nonsmoker with normal blood pressure, a normal BMI, an LDL-cholesterol level of 95 mg/dL, an HDL-cholesterol level of 50 mg/dL, and a triglyceride level of 195 mg/dL
  2. A 45-year-old female nonsmoker with a normal blood pressure and BMI, an LDL-cholesterol level of 120 mg/dL, an HDL-cholesterol level of 56 mg/dL, and a triglyceride level of 105 mg/dL
  3. A 35-year-old obese male smoker with prehypertension, an LDL-cholesterol level of 125 mg/dL, an HDL-cholesterol level of 39 mg/dL, and a triglyceride level of 125 mg/dL
  4. A 72-year-old normotensive male nonsmoker with a BMI of 23.1 kg/m2, a past history of stroke, an LDL-cholesterol level of 65 mg/dL, an HDL-cholesterol level of 50 mg/dL, and a triglyceride level of 145 mg/dL
  5. A 65-year-old female nonsmoker with a normal blood pressure and BMI, an LDL-cholesterol level of 80 mg/dL, an HDL-cholesterol level of 50 mg/dL, and a triglyceride level of 294 mg/dL

Critique:

The American Diabetes Association (ADA) recommends that for patients of all ages with diabetes and atherosclerotic cardiovascular disease, high-intensity statin therapy should be added to lifestyle therapy (SOE A).

For patients with diabetes who are less than 40 years of age and have additional risk factors for atherosclerotic cardiovascular disease, the ADA recommends that patient and provider consider moderate-intensity statin therapy in addition to lifestyle therapy (SOE C).

For patients with diabetes who are 40–75 years of age (SOE A) or >75 years of age (SOE B) WITHOUT atherosclerotic cardiovascular disease, the ADA favors the use of moderate-intensity statin therapy in addition to lifestyle therapy.